SlideShare a Scribd company logo
Chapter 4: The Inflammatory Response Study Mode
1. How many cases of severe sepsis occur in the United States each year?
A. 250,000
B. 500,000
C. 750,000
D. 1 million
2. Cytokines are:
A. Lipid mediators secreted by lymphocytes
B. Proteins that form the contractile elements of muscle cells
C. Toxins secreted by bacteria
D. Small hormone-like proteins secreted by individual cells
3. Which of the following mediators is a Th2 cytokine?
A. TNF
B. IL-1β
C. IL-10
D. IL-12
4. Which of the following receptors is essential for activation of macrophages by bacterial
lipopolysaccharide (LPS)?
A. TLR4
B. IL-1RI
C. IRAK
D. LBP
5. Treatment with a monoclonal anti-TNF antibody has been shown to be effective for the
treatment of selected cases of which of these diseases?
A. Asthma
B. Septic shock
C. Myocardial infarction
D. Crohn's disease
6. Which of the following agents has been approved by the FDA for the adjuvant
treatment of severe sepsis?
A. Recombinant human IL-6
B. Recombinant human activated protein C
C. Recombinant human growth hormone
D. Recombinant human IL-1RA
7. In critically ill victims of trauma, high circulating levels of IL-6 are:
A. Associated with an increased risk of death
B. Associated with a high likelihood of survival
C. Rarely detectable
D. A measurement artifact
8. Which of the following is the main cell type that is activated by IL-8?
A. Enterocytes
B. Macrophages
C. Monocytes
D. Neutrophils
9. HMGB1 is:
A. A late-acting proinflammatory cytokine
B. A nuclear protein that supports DNA transcription
C. A protein with high electrophoretic mobility
D. All of the above
10. Nitric oxide:
A. Is a widely used anesthetic gas
B. Is a potent endogenous vasoconstrictor
C. Is generated by cells from the amino acid L-arginine
D. Stimulates the aggregation of platelets
Chapter 06: Metabolism in Surgical Patients
1. A 5-year-old boy with an unresolved congenital umbilical hernia is admitted for
hernioplasty. After a moderate fasting period prior to surgery, the child is profoundly
asleep and unable to be roused. Marked hypoglycemia and ketonuria, accompanied by low
levels of alanine and insulin, are noted during workup. Administration of alanine produces
a rapid rise in his blood glucose level. The metabolic alteration most likely causing the
symptoms of this patient is expected in which of the following pathways?
A. Protein breakdown in muscle tissue
B. Mitochondrial β-oxidation of fatty acids by the liver
C. Lipolysis by desnutrin in adipose tissue
D. Glucagon secretion by alpha cells of the pancreas
2. A 52-year-old man is admitted with an episode of severe upper gastrointestinal bleeding.
The patient has a 25-year history of excessive alcohol consumption and was diagnosed with
cirrhosis 5 years ago. Because of recurrent and refractory bleeding from esophageal
varices, he consents to an emergency portocaval shunt. After surgery, the bleeding is
controlled and the patient seems to be improving until he subsequently falls into a deep
coma. The best next step in the diagnostic assessment of this patient is to order which of the
following tests?
A. Blood glucose level
B. Serum bilirubin level
C. Serum albumin level
D. Blood culture
E. Serum ammonia level
3. Indicate which of the following is not a contraindication to enteral nutrition.
A. Gastrointestinal ischemia
B. Severe short bowel syndrome
C. Distal high-output intestinal fistulas
D. Severe acute pancreatitis
4. In the management of enteral feeding associated diarrhea, which of the following is the
most appropriate initial action?
A. Change of antibiotics in use.
B. Change to an elemental-type formula (e.g., EleCare).
C. Change to an enteral formulation with fiber.
D. Start a small dose of loperamide.
5. In performing perioperative assessments, the most reliable biochemical predictors of
operative morbidity and mortality across surgical specialties include which of the
following?
A. Serum albumin level
B. Defects in cellular immunity and phagocytic function
C. Serum urea nitrogen level
D. Prothrombin time
6. A patient receives 2500 mL of Vivonex Pediatric/day, consisting of a total of 60 g of fat,
60 g of protein, and 325 g of carbohydrates. The daily caloric intake of this patient is best
represented by which of the following values?
A. 1600 kcal/day
B. 1700 kcal/day
C. 1900 kcal/day
D. 2200 kcal/day
7. A previously healthy, 18-year-old man is admitted to the ICU after sustaining multiple
injuries in a motorcycle accident. Multiple facial fractures, as well as ribs, pelvis, and right
femur fractures, were found in the initial scan. In addition, multiple deep dermal abrasions
affecting 40% of his body surface area were apparent. Nutritional support should be
initiated by which of the following?
A. Preferentially via the IV route
B. Only via enteral feeding tube to decrease the patient's high risk of aspiration
C. Immediately after resuscitation is complete (i.e., within the first 48 to 72
hours)
D. After definitive treatment of the injuries has been completed
8. This question concerns the same patient as in question 7.) Two months postinjury and
after operative procedures that included tracheostomy, open reductions with internal
fixation, exploratory laparotomy, and persistent ventilator support, a 20% loss of body
weight is noted as compared with admission. By simply considering weight loss, which of
the following can be accurately expected?
A. 50% increased risk of mortality
B. Reduced risk for pressure sores and pneumonia
C. 10% increased risk of mortality
D. Increased difficulty healing and weaning of ventilator support
9. A 9-year-old girl suffered an 86% TBSA, third-degree burn injury during a house fire.
She was found unconscious. The patient arrives to the burn unit with a heart rate (HR) of
130 beats/min, BP of 100/70 mm Hg, respiratory rate (RR) of 18 breaths/min, and
temperature (T) of 37.6° C. She receives standard care with adequate IV resuscitation,
thermoregulation of the room's thermostat to 33° C, and excision and grafting of her
wounds on postinjury day. What is the most reliable method to estimate caloric
requirements in this patient? (Questions 9 through 12 apply to this patient.)
A. Harris-Benedict equation
B. Curreri formula
C. Indirect calorimetry
D. Fick's equation
10. 7 days postinjury, HR = 165 beats/min, BP = 105/80 mm Hg, RR = 22 breaths/min, T =
38.5° C, plasma glucose level = 250 mg/dL, [Na] = 145 mEq/dL, [Cl] = 100 mEq/dL, and
[K] = 4.5 mEq/dL. You administer IV fluids, insulin, and potassium chloride. What should
be done to prevent mortality through the management of hyperglycemia in this critically ill
surgical patient?
A. Titrate patients to a glucose level goal between 80 and 110 mg/dL.
B. Consider starting the patient on an oral hypoglycemic.
C. Use insulin only if the plasma glucose level is >200 mg/dL
D. Minimize significant fluctuations of glucose and potassium carefully.
E. Request an endocrinology consultation.
11. Fifteen days following severe burns, excision, and autograft surgery, an increase of
>25% of insulin requirements is noted over the previous 24-hour period. What is the best
next step in the management of this patient?
A. Schedule further surgery to decrease hypermetabolic response.
B. Further increase the insulin drip until a glucose level of 140 to 180 mg/dL
is reached.
C. Order cultures and band neutrophil of peripheral blood.
D. Repeat blood glucose level testing and order a new metabolic panel.
12. Four weeks postinjury, during the daily abdominal examination, you palpate the lower
edge of the liver 4 cm below the edge of the ribs. No tenderness to palpation is reported and
there is no evidence of jaundice. Which of the following best explains the findings seen in
pathology?
A. Increased dietary intake of fats
B. Increased synthesis of fats from sugar-enriched diet
C. Excessive peripheral lipolysis.
D. Decreased β-oxidation of fat in liver mitochondria
Chapter 07: Wound Healing
1. Fetal wound healing is different from adult wound healing in that:
A. Fetal wounds heal without scarring and without dermal appendages.
B. Fetal wounds have increased amounts of TGF-β and FGF-2.
C. Fetal fibroblasts have decreased prolyl hydroxylase activity.
D. Fetal wounds have increased responses to inflammation and growth factor stimulation.
E. The ECM of the fetal wound has low levels of hyaluronic acid.
2. Elastin is:
A. Organized in mammalian skin in a basket weave pattern to resist
multidimensional tensile stress
B. Produced late in life; has a high turnover rate
C. An extremely hydrophilic molecule, which accounts for its functional
properties
D. An important component of the extracellular matrix of blood vessels;
mutations causing elastin protein deficiency result in intimal hyperplasia,
leading to arterial narrowing.
E. Affected in Ehlers-Danlos syndrome, which is characterized by fragile skin
3. The cells or cell components central to wound healing are:
A. B cells
B. T cells
C. Leukocytes
D. Macrophages
E. Platelets
4. Thromboxane causes:
A. Fibroblast chemotaxis
B. Vasoconstriction
C. Collagen cross linking
D. Endothelial proliferation
E. Bacterial lysis
5. Iron deficiency has an impact on wound healing by decreasing:
A. Early tensile strength
B. DNA synthesis
C. Conversion of hydroxyproline to proline
D. Tissue oxygenation
E. Fibroblast proliferation
6. Hypertrophic scar is:
A. Another term for keloids
B. More likely to occur on the face
C. Genetic in origin
D. Preventable
E. Worsened with glucocorticoids
7. The effects of diabetes on wound healing include:
A. Slowed epithelialization
B. Reduced phagocytosis
C. Glycosylated collagen
D. Thickened basement membrane
E. All of the above
8. Ionizing radiation causes hypoxia by:
A. Direct cellular injury to endothelium
B. Basal membrane injury
C. Release of histamine and serotonin
D. Preventing the hypoxic stimulus of angiogenesis
E. Increased dermal fibrosis and thickening
9. Nicotine ingestion affects wound healing by:
A. Increasing fibroblast proliferation
B. Increasing platelet adhesion
C. Competitively competing with oxygen
D. Inhibiting oxidative metabolism
E. Inhibiting oxygen transport
10. Which of the following events occurs in the proliferative phase of wound healing?
A. Histamine release
B. Collagen cross linking
C. Thromboxane release
D. Phagocytosis
E. Collagen synthesis
11.Chronic wounds characteristically have:
A. Tissue inflammation
B. Decreased tissue inhibitor of metalloproteinases levels
C. Increased gelatinase levels
D. Increased collagenase levels
E. All of the above
12. The wound healing impairment caused by corticosteroid administration can be
reversed by:
A. Vitamin A
B. Vitamin C
C. Zinc
D. Vitamin K
E. Vitamin B12
13. Which of the following glycosaminoglycans is not a component of skin?
A. Hyaluronic acid
B. Chondroitin sulfate
C. Dermatan sulfate
D. Heparin sulfate
E. Heparin
14. Most human collagen is:
A. Type I
B. Type III
C. Type IV
D. Type V
E. Type VII
15. Endothelial cells are induced to form tubules by:
A. Vascular endothelial growth factor (VEGF)
B. Hypoxia
C. Tumor necrosis factor-α (TNF-α)
D. Transforming growth factor-β (TGF-β)
Chapter 08: Regenerative Medicine
1. Which of the following is not a primary cellular source currently being investigated for
use in tissue repair?
A. Embryonic stem cells
B. Somatic cell nuclear transfer
C. Circulating fetal stem cells
D. Stromal fraction of adult bone marrow and fat
E. Cancer stem cells
2. Adult mesenchymal stem cells can be characterized by:
A. Their ability to undergo clonal expansion, with the ability to differentiate
into fat, cartilage, and bone under appropriate conditions
B. Their low frequency in fat, but significantly higher frequency and ease of harvest in bone
marrow, with minimal morbidity
C. The disparate growth kinetics and gene transduction capacity between fat and bone marrow
sources
D. The inability for bone marrow–derived cells to undergo myogenic differentiation
E. A higher risk for whole blood contamination from fat-derived cells relative to bone marrow
sources
3. Which of the following is not true of fetal stem cells?
A. Fetal stem cells do not proliferate as fast as adult stem cells.
B. Fetal stem cells have been found to possess capacity for adipogenic, osteogenic, and
chondrogenic differentiation.
C. Xenogeneic transplantation has shown fetal stem cells to engraft and undergo site-specific
tissue differentiation.
D. The use of fetal stem cells is limited by ethical debate and attendant risks associated with
intrauterine procedures.
4. Which of the following is not one of the transcription factors used in cellular
reprogramming to create iPS cells?
A. Oct-4
B. Sox-9
C. Klf-4
D. Nanog
5. Which of the following is not true of ASCs?
A. ASCs can be differentiated into bone, fat, and cartilage.
B. The major advantage of ASCs is their relative abundance and ease of
isolation from subcutaneous adipose tissue through standard lipoaspirate
techniques.
C. ASCs represent a homogeneous cell line derived from lipoaspirate cells.
D. None of the above
6. Which of the following cell types is not pluripotent?
A. ESCs
B. iPS cells
C. ASCs
D. Cells derived by somatic cell nuclear transfer (SCNTs)
7. Induced pluripotent stem cells are characterized by:
A. Requirement for viral integration of defined transcription factors to dedifferentiate into
pluripotent state
B. Cells that are identical to embryonic stem cells
C. Cells that give rise to teratoma comprising all three germ layers when
injected into immunodeficient mouse
D. Inability to differentiate into neurons
8. In the skin, epidermal stem cells reside in:
A. Sweat glands
B. Bulge region along hair follicles
C. Superficial epidermis
D. Subcutaneous fat
Chapter 09: Evidence-Based Surgery: Critically
1. The difference between efficacy and effectiveness is:
A. The manner in which the outcome of interest is measured.
B. Efficacy is based on best case patient care and outcomes from research settings, whereas
effectiveness more closely approximates real-world results.
C. Effectiveness is based on best case patient care and outcomes from research settings, whereas
efficacy more closely approximates real-world results.
D. No difference.
2. In performing a health economic analysis, the author(s) must be certain to describe
which of the following?
A. The perspective being adopted
B. Discounting (to account for the future value of the dollar, usually 3% to 5%) and inflation
adjustment
C. The assessment of costs rather than charges
D. All of the above
3. Using observational data, causality can be inferred if:
A. Exposure precedes the outcome.
B. The exposure can plausibly and/or biologically lead to the outcome.
C. The magnitude of the association between exposure and outcome is large, with potentially
varying magnitudes of association between exposure dose and outcome.
D. All of the above.
4. In any study, both the hypothesis and plan of analysis should be stated a priori to
decrease the risk of:
A. A negative study.
B. A type I error.
C. A type II error.
D. Low statistical power.
5. A group of investigators perform a randomized trial comparing a control intervention
(A) to an experimental intervention (B). Their hypothesis is that intervention B is superior
to A with regard to the outcome of interest. The trial is adequately powered to identify a
difference if one truly exists. However, the results demonstrate no statistically significant
difference between these two interventions. The investigators conclude that the two
interventions are likely equivalent with regard to this outcome. What can safely be stated
about the result and the authors' conclusions?
A. The authors are correct; intervention B is equivalent to A in terms of the outcome of interest.
B. Intervention B would likely demonstrate a statistically significant improvement over A if the
sample size were larger.
C. The authors are incorrect because equivalence can only be ascertained using a noninferiority
trial design.
D. The conclusions depend on the manner in which the authors chose to control for confounding
factors.
6. Which of the following is true regarding the commonly used P value significance level of
.05?
A. This has been mathematically shown to be the cutoff for statistical significance.
B. It is commonly used and should therefore always be chosen as the cutoff for statistical
significance.
C. If a statistical test attains this level of significance, it definitively proves a difference exists.
D. It is arbitrary and a different level for statistical significance can be selected.
7. In analyzing data from a randomized trial, which of the following analytic approaches is
most appropriate?
A. Per-protocol
B. Case-complete
C. Intent to treat
D. Meta-analysis
8. Which of the following is true when comparing the odds with the probability of an
outcome?
A. The odds can overestimate the probability if the outcome of interest is common in the
underlying population.
B. The odds can underestimate the probability if the outcome of interest is common in the
underlying population.
C. The odds can overestimate or underestimate the probability if the outcome of interest is
common in the underlying population.
D. The two values are the same.
Chapter 13: Surgical Complications
1. Which of the following are mechanisms for heat loss that contribute to the development
of hypothermia?
A .Cool environment
B. Direct body contact to cooler materials
C. Heat loss with evaporated water vapor
D. Exhalation of warmed air
E. All the above
2. Which of the following is required to make a definitive diagnosis of malignant
hyperthermia?
A. Administration of an epidural anesthetic
B. Tachycardia
C. Cyanosis
D. Muscle biopsy
E. Muscle rigidity
3. Which of the following is not usually required for the clinical diagnosis of pneumonia?
A. Decreased breath sounds
B. Temperature that is usually greater than 38.5°C
C. Pleuritic chest pain with coughing
D. Elevated peripheral white blood cell count
E. Chest radiographic infiltrate
4. Which of the following criteria is indicative of the presence of acute lung injury (ALI)
rather than adult respiratory distress syndrome (ARDS)?
A. Pulmonary capillary wedge pressure < 20 mm Hg
B. PaO2/FIO2 ratio < 300
C. PaO2/FIO2 ratio < 200
D. Bilateral infiltrates on chest radiography
E. An acute change in lung function
5. What is the expected reinfarction rate for patients undergoing noncardiac surgery after
a recent acute myocardial infarction (AMI)?
A. No difference compared with other myocardial infarction patients
B. 8% at less than 3 months, 3.5% at 3 to 6 months, then similar to other myocardial infarction
patients
C. 20% in the first 6 months and then similar to other myocardial infarction patients
D. 15% at less than 3 months, 10% at 3 to 6 months, 5% at more than 6 months
E. 50% at less than 3 months, 25% at 3 to 6 months, 15% at more than 6 months
6. Components of the syndrome of inappropriate secretion of antidiuretic hormone
(SIADH) include:
A. Hyponatremia
B. Hypernatremia
C. Peripheral edema
D. Serum hyperosmolality
E. Hypertension
7. Which of the following is not a common cause of a small bowel obstruction in an adult?
A. Hernia
B. Tumor
C. Volvulus
D. Adhesions
E. Ileocolic intussusception
8. An abdominal compartment syndrome produces all the following except:
A. Acute renal failure
B. Hypoxia
C. Intestinal obstruction
D. Elevated urinary bladder pressure
E. Hypercarbia
9. Intital treatment of acute gastrointestinal bleeding includes:
A. H2 receptor antagonists
B. Aggressive volume resuscitation
C. Gastrointestinal endoscopy
D. Sucralfate
E. Antibiotics
10. Surgical antibiotic prophylaxis is indicated:
A. In all emergency operations
B. For wounds classified as clean-contaminated
C. For wounds classified as contaminated
D. For wounds classified as dirty-infected
E. For all wounds
11. Postrenal causes of acute renal failure include all the following except:
A. Ureteral obstruction caused by stones
B. Bladder dysfunction caused by nerve injury
C. Urethral obstruction caused by prostatic enlargement
D. A blocked Foley catheter
E. Myoglobinuria
Chapter 14: Surgery in the Geriatric Patient
1. Which of the following statements is true?
A. Life expectancy of a patient 90 years old, who has no comorbid conditions, is 6 months.
B. The proportion of individuals 65 years old and older will remain constant over the next 50
years.
C. Age is frequently a risk factor in predicting postoperative morbidity and mortality.
D. There is a decline in physiologic function in all organ systems with aging.
E. It is estimated that approximately 25% of patients in most general surgical practices are older
than 65 years.
2. Which of the following statements is false?
A. Systolic cardiac function decreases with age at a constant rate.
B. Predictable decreases in renal function occur with age and relate specifically to the glomerular
filtration rate.
C. Insulin secretion from the beta cell decreases as a function of age.
D. Forced vital capacity (FVC) and forced expiratory volume in 1 second (FEV1) decrease
steadily with aging.
E. The synthetic capacity of the liver, as measured by standard liver function tests, remains
unchanged with age.
3. Which of the following is not a necessary part of the preoperative assessment in an older
adult?
A. Baseline cognitive evaluation
B. Exercise capacity testing
C. Determination of nutritional status
D. Advance directives and discussion of end-of-life wishes
E. Evaluation of ADLs and IADLs
4. In emergency surgery for perforated ulcer disease, a patient with preoperative shock,
more than 48 hours of perforation, and significant comorbid disease has a mortality rate
close to:
A. 0%
B. 25%
C. 50%
D. 75%
E. 100%
5. Which of the following is true regarding appendicitis in older adults ?
A. Most patients present with classic signs and symptoms of right lower quadrant pain, increased
white blood cell count, and fever.
B. Although there are typically delays in presentation to the hospital, the diagnosis is usually
made in a timely fashion.
C. Of these patients, 18% present with no abdominal pain.
D. Reported rates of perforated appendicitis in octogenarians are less than 50%.
E. The overall mortality in patients older than age 65 with appendicitis is approximately 18%.
6. In an older patient with suspected biliary disease, which of the following statements is
false?
A. The rate of gallstones is 30% to 40% of individuals older than 80 years.
B. There is an increased incidence of common bile duct stones in patients undergoing
cholecystectomy.
C. Gallbladder motility is typically normal.
D. The conversion rate from laparoscopic cholecystectomy to open procedures is higher than in
younger patients.
E. Biliary tract disease is a frequent cause of acute abdominal complaints in older adults.
7. Which of the following statements is not true?
A. Breast cancer in older women is more frequently associated with the presence of favorable
tumor markers.
B. Stage per stage, survival for older women with breast cancer is better than that seen in
younger women.
C. Breast cancer trials in the United States have a disproportionately low enrollment of older
women.
D. Advanced age is not a contraindication to breast-conserving surgery.
E. older women have an increased incidence of significant side effects from axillary lymph node
dissection (ALND).
8. Which of the following is not currently recommended for an older surgical patient?
A. Carotid endarterectomy
B. Carotid angioplasty and stenting
C. Open abdominal aortic aneurysm repair
D. Endovascular aneurysm repair
E. Peripheral arterial bypass
Chapter 15: Morbid Obesity
1. Hormones or peptides involved in satiety include:
A. Gastrin
B. Somatostatin
C. Glucagon
D. Ghrelin
E. Estrogen
2. Currently accepted guidelines from the National Institutes of Health for preoperative
selection of patients for weight reduction surgery include all the following except:
A. Patients with Prader-Willi syndrome
B. BMI > 35 kg/m2
with associated medical comorbidity worsened by obesity
C. Failed medical therapy
D. Psychiatrically stable
E. Motivated patient
3. Absolute contraindications for bariatric surgery include:
A. Cardiomyopathy
B. Pickwickian syndrome
C. Type 1 diabetes mellitus
D. Nonalcoholic steatotic hepatitis
E. None of the above
4. Laparoscopic sleeve gastrectomy has:
A. A higher leak rate than laparoscopic RYGB
B. A lower mortality rate for super obese BMI > 60 kg/m2
undergoing duodenal switch
C. A theoretical advantage over RYGB for iron absorption
D. All of the above
E. None of the above
5. One specific problem that may arise with persistent vomiting after any of the bariatric
operations is Wernicke's encephalopathy, which can be treated with parenteral:
A. Vitamin B12
B. Omeprazole
C. Thiamine (vitamin B1)
D. Ascorbic acid (vitamin C)
E. Scopolamine
6. Morbid obesity is defined as:
A. 1.5 times ideal body weight
B. BMI > 40 kg/mg2
C. 20% above ideal body weight for adolescents
D. A function of physical activity, comorbid conditions, and weight
E. Weight > 40 kg
7. Bariatric operative procedures that produce weight loss by a combination of restriction
of oral intake and malabsorption include all the following except:
A. Vertical banded gastroplasty
B. Duodenal switch
C. Biliopancreatic diversion
D. Roux-en-Y gastric bypass
E. None of the above
8. Long-term metabolic complications of Roux-en-Y gastric bypass include:
A. Hyperlipidemia
B. Vitamin C deficiency
C. Vitamin K deficiency
D. Lactic acidosis
E. Iron deficiency
9. Medical therapy that has proven to have significant long-term success in morbidly obese
patients includes:
A. Low-calorie diets
B. Sibutramine
C. Ghrelin
D. Orlistat
E. None of the above
10. Produced in the proximal stomach, levels of the hormone ghrelin:
A. When decreased, seem to produce increased food intake
B. Are suppressed in postoperative patients who have undergone gastric bypass
C. Are decreased in individuals on a low-calorie diet
D. Regulate release of insulin
E. Are decreased in patients after adjustable gastric banding.
Chapter 16: Anesthesiology Principles, Pain
1. Which of the following statements is true regarding isoflurane compared with
halothane?
A. Isoflurane is associated with smoother inhalational induction.
B. Isoflurane produces greater sensitization to the arrhythmogenic effects of catecholamines.
C. Isoflurane has greater potency, as reflected in a lower minimal alveolar concentration
(MAC).
D. Isoflurane is associated with more rapid emergence.
E. Isoflurane increases bronchoconstriction, whereas halothane decreases
bronchoconstriction.
2. Which of the following drugs is useful as a premedicant because of its potent amnesic
effects?
A. Glycopyrrolate
B. Etomidate
C. Midazolam
D. Ketamine
E. Thiopental
3. Which of the following statements most accurately describes the differences between
subarachnoid block and epidural block?
A. Subarachnoid block is associated with more rapid onset of hypotension.
B. Subarachnoid block is associated with greater risk of systemic local anesthetic toxicity.
C. Subarachnoid block is associated with less risk of post–lumbar puncture headache.
D. Subarachnoid block should not be performed with a mixture of local anesthetic and opioids.
E. Subarachnoid block is associated with a small risk of cardiac arrest for which resuscitation is
uncomplicated.
4. Essential monitors for all anesthetics include which of the following?
A. Direct arterial pressure
B. Exhaled volatile anesthetic concentration
C. Bioimpedance cardiac output
D. Electrocardiography
E. Anesthetic depth (bispectral index)
5. Which of the following muscle relaxants is largely metabolized by Hofman degradation
in plasma and is relatively independent of renal elimination?
A. Pancuronium
B. Vecuronium
C. Cisatracurium
D. Rocuronium
E. Atracurium
6. Which of the following statements is accurate regarding preoperative cardiac
evaluation?
A. Older patients undergoing cataract surgery require preoperative stress testing.
B. Patients who require major vascular surgery should undergo cardiac catheterization before
scheduling the vascular procedure.
C. Patients can be stratified for the need for cardiac evaluation based on symptoms and
magnitude of the anticipated surgery.
D. Patients with previous myocardial revascularization are at high risk for perioperative
myocardial infarction.
E. Ambulatory electrocardiography is sensitive and specific for the identification of patients at
high perioperative risk.
7. Which of the following guidelines is recommended for preoperative fasting?
A. Aspiration of gastric contents is not a problem; all patients can take food and liquids freely
until immediately before surgery.
B. A fasting period of 2 hours or more is recommended after ingestion of clear liquids.
C. All patients should not receive food or liquids after midnight before surgery the next day.
D. A fasting period of 6 hours or more is recommended after ingestion of solids.
E. Both B and D
8. Which of the following statements is accurate regarding local anesthetic toxicity?
A. The earliest symptoms are referable to the central nervous system.
B. At the first sign of local anesthetic toxicity, succinylcholine should be given.
C. When local anesthetics are used for regional block, adding epinephrine decreases the toxic
dose.
D. Bupivacaine is the least toxic of currently used local anesthetics.
E. Ester-type local anesthetic agents are more toxic than amide agents because of slow
metabolism.
9. Which of the following problems is commonly recognized in the postanesthesia care unit
(PACU)?
A. Delirium
B. Emesis
C. Hypoxemia
D. Hypertension
E. All of the above
10. Characteristics of moderate sedation include which of the following?
A. Absence of movement in response to a skin incision
B. Preserved airway reflexes
C. Motor response only to painful stimuli
D. Moderate respiratory depression
E. Moderate hypotension
11. Which four steps accurately describe the process of nociception?
A. Transduction, transmission, modulation, and perception
B. Recognition, registration, amplification, and interpretation
C. Perception, integration, orientation, and implementation
D. Description, analysis, formulation, and recognition
E. Deformation, translation, registration, and formulation
12. Which of the following opioids is partially converted to a metabolite that can
accumulate and cause seizures in patients with renal impairment?
A. Fentanyl
B. Hydromorphone
C. Codeine
D. Morphine
E. Meperidine
13 .What is the correct term for the physiologic process in which a previously effective dose
of an opioid fails to provide adequate analgesia?
A. Addiction
B. Psychological dependence
C. Physical dependence
D. Tolerance
E. Malingering
14. Compared with patients who receive conventional, intermittent, nurse-administered
opioid delivery, patients who receive intravenous patient-controlled analgesia experience
which of the following advantages?
A. Prompt analgesia
B. Smaller doses of opioids
C. Better maintenance of blood concentration of drugs in the analgesic range
D. Lower incidence of drug-related side effects
E. All of the above
Chapter 19: The Difficult Abdominal Wall
1. Abbreviated laparotomy is the initial phase of damage control surgery. The indications
are as follows:
A. Temperature less than 35° C
B. Medical bleeding
C. Arterial pH less than 7.20
D. Urine output less than 30 mL/kg/hr
E. A, B, and C are correct
2. The most common indications for the use of the open abdomen technique in general
surgery are as follows:
A. Abdominal compartment syndrome
B. Ruptured abdominal aortic aneurysm
C. Trauma-damage control
D. Acute pancreatitis
E. All of the above
3. If not recognized and treated, intra-abdominal hypertension can progress to abdominal
compartment syndrome. Which of the following clinical signs are hallmarks of abdominal
compartment syndrome?
A. Oliguria
B. Metabolic alkalosis
C. Increased peak inspiratory pressures
D. Intestinal ileus
E. A and C are correct
4. There are several techniques for creating a temporary abdominal closure for the open
abdomen. The key to all techniques must include the following:
A. Quick application
B. Seal in moisture and temperature
C. Quickly removable
D. High tensile strength
E. A, B, and C are correct
5. During the staged abdominal repair phase of damage control surgery, the surgeon has
several challenging questions to answer on return to the operating room. Which of the
following approaches can be used to address small and large bowel injuries?
A. Resection of devitalized tissue
B. Primary repair
C. Externalization with creation of a stoma
D. Primary bowel anastomosis
E. All of the above
6. The open abdomen technique has a high rate of nonclosure because of the following
complications:
A. Intra-abdominal abscess and intra-abdominal sepsis
B. Acute lung injury
C. Atmospheric intestinal fistula
D. Urinary tract infection
E. A and C are correct
7. Which of the following mesh products should not be used in the open abdomen setting
because they have very high rates of intestinal fistula formation and mesh infection?
A. Human dermal acellular dermis
B. PTFE
C. Porcine dermal matrix
D. Polypropylene
E. B and D are correct
8. The decision to close an open abdomen with visceral edema can be complicated. Which
of the following physiologic criteria can be used to guide abdominal closure in the
operating room?
A. Decrease in urine output by 10 mL/hr
B. Change in peak inspiratory airway pressure less than 10 cm H2O while attempting to bring the
fascia together
C. Increase in intracranial pressure by 5 cm H2O
D. Sustained increase in intra-abdominal pressure less than 20 mm Hg
E. B and C are correct
Chapter 20: Emergency Care of Musculoskeletal
1. Which of the following will increase the stiffness of an external fixation construct?
A. Using stainless steel instead of titanium pins
B. Using more pins
C. Placing the bars closer to the bone
D. Placing the bars in multiple planes
E. All of the above
2. A patient is found to have an isolated fracture of the medial malleolus on an ankle series
x-ray. What other imaging should be performed?
A. AP, lateral, and oblique views of the foot to look for a fifth metatarsal fracture.
B. CT scanning of the ankle to look for a tibial plafond fracture
C. AP and lateral views of the tibia and fibula to look for a proximal fibular fracture
D. AP and cross-table lateral views of the hip to look for a femoral neck fracture
E. PA and lateral views of the lumbar spine to look for a lumbar burst fracture
3. An absolute indication to perform a four-compartment fasciotomy of the leg for
compartment syndrome is:
A. Firm compartments on physical examination
B. ∆P < 30 mm Hg
C. Subjective complaints of paresthesias in the foot
D. Severe leg pain
E. Unconscious patient with a tibial shaft fracture
4. A 36-year-old man presents to the trauma bay after a motorcycle crash. His systolic
pressures remain in the 70s despite resuscitation with packed red blood cells and
crystalloid. His chest x-ray is normal. His pelvic x-ray is shown in Figure 20-30. What is the
next most appropriate treatment?
A. Application of a pelvic binder
B. Placement of a chest tube
C. CT of chest, abdomen, and pelvis
D. Emergent exploratory laparotomy
E. Arteriography
5. A 26-year-old man presents to the emergency department with the fracture seen in
Figure 20-4A. A Hare traction splint was placed in the field. Traction on the injured limb
should be maintained. Which of the following is the most appropriate method of traction
for this patient?
A.Skin traction with a Buck boot
B. Distal femoral traction pin placed from medial to lateral
C.Proximal tibial traction pin placed from medial to lateral
D.Proximal tibial traction pin placed from lateral to medial
E. Hare traction splint left in place
6. A 47-year-old woman sustains the fracture shown in Figure 20-40. Which examination
tests the nerve most commonly injured with this fracture pattern?
A. Shoulder abduction
B. Elbow flexion
C. Wrist extension
D. Wrist flexion
E. Finger flexion
Chapter 21: Burns
1. The zone of stasis in a burn wound is associated with which of the following?
A. Direct thermal damage
B. Vasodilation
C. Neutrophil adherence
D. Platelet degranulation
E. Non-nutrient shunting
2. Deep second-degree wounds reepithelialize from retained keratinocytes in:
A.Rete ridges
B. Hair follicles
C. Moll glands
D. Reticular dermis
E. Meissner corpuscles
3. A patient with burns to the entire back, scalp (50% of the head and neck), and posterior
thighs has what percentage of his or her total body surface area (TBSA) burned?
A.40%
B.28%
C.20%
D.32%
E.36%
4. Severe burns are associated with which of the following immunodeficiencies in the acute
phase?
A. Neutropenia
B. Granulocyte colony-stimulating factor deficiency
C. Decreased cytotoxic T cell activity
D. Increased neutrophil apoptosis
E. Antibody overproduction
5. After major burn injury, the metabolic changes are characterized by an ebb and flow
phase. Changes consistent with the ebb phase include:
A. Increased oxygen delivery
B. Low cardiac output
C. Hypermetabolism
D. Hyperthermia
E. Hyperglycemia
6. Which of these therapeutic approaches to attenuate the hypermetabolic response can
lead to hyperglycemia?
A. Insulin
B. Growth hormone
C. Metformin
D. PPAR-γ agonists
E. Oxandralone
7. A 40-year-old, 100-kg man is involved in a house fire with burns to 45% of his TBSA.
He comes to the emergency department with two peripheral IV lines that are not being
used. It is 2 hours since his injury, and he has not received any resuscitation. His initial IV
fluid rate should be:
A. 250 mL/hr
B. 500 mL/hr
C. 1000 mL/hr
D. 1500 mL/hr
E. 2000 mL/hr
8. The relative surface area of regions of the body used in calculating burn surface area
differs in children from adults. Which of the following statements describes these
differences?
A. Children have relatively less surface area in the arms and legs and more in the trunk.
B. Adults have relatively more surface area in the trunk and less in the upper extremities.
C. Children have relatively more surface area in the head and neck and less in the lower
extremities.
D. Adults have more surface area in the head and neck and less in the lower extremities.
E. Children have relatively less area in the head and neck and more in the trunk
9. Which of the following synthetic and biologic dressings are currently used for burn
wound closure?
A. Allograft
B. Xenograft
C. Stem cells
D. Biobrane
E. Integra
10. Injury from smoke inhalation during a house fire occurs from which of the following?
A. Thermal injury
B. Excessive coughing
C. Splinting leading to atelectasis
D. Plugging of airways from concentration of soot
E. Toxic chemicals in smoke particles
11. What are the three zones of injury after burn?
A. Coagulation, stasis, necrosis
B. Fibrinolysis, stasis, injury
C. Coagulation, injury (stasis), hyperemia
D. Edema, injury, necrosis
12. What proinflammatory mediators are responsible for the postburn hypermetabolic
response?
A.TNF-α, IL-1, IL-6, catecholamines, glucagon, cortisol, endotoxin, nitric oxide
B. Cortisol, IL-5, IL-2, epinephrine, insulin
C. IL-10, CD4, insulin, TNF-α, glucosamine
D. Glucose, cortisol, norepinephrine, nitric oxide
13. One cause of multisystem organ failure after severe burn injury is:
A. Decreased intestinal permeability to macromolecules
B. Diminished blood volume and cardiac output
C. Decreased peripheral vascular resistance
D. Decreased presence of endotoxin
14. Severely burned patients with no other complications can lose 25% of total body mass
after acute burn injury. This loss is associated with:
A. Pneumonia and pressure ulcers
B. Pneumonia and decreased wound healing
C. Immune dysfunction and death
D. Decreased wound healing and immune dysfunction
15. Improvements in morbidity from severe burn injury stem from:
A. Decreased ambient temperature, late excision and grafting after resuscitation, and early
aggressive parenteral nutrition
B. Increased ambient temperature, early excision and grafting, and early enteral nutrition
C. Controlling sepsis and late excision and grafting
D. Selective beta blockers and high-fat and high-protein diets
Chapter 22: Bites and Stings
1. Which of the following statements regarding snakebite management in North America is
true?
A. Antivenom should be administered to any patient who presents to the hospital with a bite
from a definitively identified rattlesnake.
B. Skin testing for possible allergy should be performed before the administration of CroFab.
C. CroFab is effective for reversing venom poisoning by all North American pit vipers and coral
snakes.
D. Fasciotomy should be performed only in the setting of objectively measured elevation of
pressures in involved muscle compartments.
2. Which of the following injuries is considered low risk for becoming infected and can be
repaired by primary closure?
A. Dog bite to the face
B. Cat bite to the upper arm
C. Human bite over the dorsum of the metacarpophalangeal joint
D. Primate bite to the foot
3. Which of the following statements is false regarding cat bites?
A. Pasteurella multocida is the primary organism involved in infections.
B. Puncture wounds are common.
C. Wound infection occurs in 10% to 20% of cat bites.
D. Most cat bites are considered high risk for infection and inappropriate for primary closure.
4. Which of the following statements is false regarding rabies?
A. Most patients acquiring rabies from a bat do not recall being in contact with the bat.
B. Rabies is caused by rhabdovirus found in the saliva of mammals.
C. Patients with preexposure rabies immunization need active immunization only.
D. If diagnosed early, rabies is usually treatable.
5. Prophylactic antibiotics should be administered for all of the following injuries except:
A. Cat bite to the foot
B. Human bite to the ear
C. Dog bite to the arm of a diabetic patient
D. Dog bite to the scalp of a child
6. Management of spider bites should include:
A. Prompt administration of antivenom for any victim of a black widow spider bite
B. Application of local cooling measures and conservative wound care
C. Empirical administration of dapsone for wounds believed to be consistent with brown recluse
spider bites
D. Early (≤1 hour) excision of the bite site to limit venom spread
7. Which of the following statements regarding Lyme disease is true?
A. Patients with early Lyme disease often present with a rash consistent with erythema
multiforme.
B. Neurologic involvement, neuroborreliosis, occurs in approximately 80% of untreated patients.
C. First-line treatment for early Lyme disease without neurologic involvement includes
doxycycline for 14 to 21 days.
D. Attempts to develop a safe effective vaccine for Lyme disease have so far proven
unsuccessful.
8. Treatment of Hymenoptera-induced anaphylactic shock should include:
A. Epinephrine
B. Antihistamines (H1 and H2 blockers)
C. Steroids
D. Referral to an allergist for possible desensitization therapy
E. All of the above
9. Treatment for a moray eel bite includes:
A. Antirabies immunization
B. Débridement and primary closure
C. Débridement and delayed primary closure
D. Administration of antivenin
10. Treatment for a sea urchin puncture includes:
A. Percussion and fragmentation of the spine
B. Rapid extraction of the spine, followed by wide excision
C. Exploration of any discolored skin marking
D. Ice water immersion
E. None of the above
Chapter 23: Surgical Critical Care
1. A patient is defined as having brain death if the patient has complete absence of cortical
brain function and which of the following?
A. Loss of pupillary reflex to light
B. Loss of the vestibulo-ocular reflex
C. Loss of oropharyngeal reflex (gag reflex)
D. Apnea on consecutive tests despite adequate stimulation (PaCO2 >60 mm Hg)
E. There is no one accepted definition of brain death nationally
2. The Confusion Assessment Method (CAM-ICU) is useful in determining the presence or
degree of which of the following in patients in the intensive care unit?
A. Chronic dementia
B. Adequacy of sedation regimens in mechanically ventilated patients
C. Adequacy of analgesia in postoperative surgical patients
D. Identifying, evaluating, and managing acute delirium
E. Managing alcohol withdrawal
3. In a mechanically ventilated patient with a traumatic brain injury or in a patient in a
coma (GCS <8) in whom there is suspected elevation of intracranial pressure (ICP), what is
an appropriate PaCO2 goal to maintain?
A. 25 to 30 mm Hg
B. 30 to 35 mm Hg
C. 35 to 40 mm Hg
D. 40 to 45 mm Hg
E. 45 to 50 mm Hg
4. Which of the following statements is true regarding measurement of central venous
pressure (CVP) in postoperative surgical patients as a guide for management of fluid
resuscitation?
A. Right-sided heart function is a reliable predictor of left-sided heart function.
B. It is inaccurate in assessing volume status in a diverse group of surgical patients.
C. Values can be interpreted regardless of the patient's degree of mechanical ventilation
requirements of positive end-expiratory pressure (PEEP).
D. It is a good guide for instituting and managing vasopressor agents in hypotensive patients.
5. A postoperative patient with new-onset unstable atrial fibrillation with systolic blood
pressure of 70 mm Hg should be treated with which of the following modalities?
A. Direct current cardioversion
B. Intravenous beta blocker
C. Intravenous calcium channel blocker
D. Digoxin
6. Which of the following modalities has been shown to reduce mortality in adult patients
with the acute respiratory distress syndrome (ARDS) in prospective randomized trials?
A. Corticosteroid early in the course of ARDS (<7 days)
B. Surfactant replacement therapy
C. Nitric oxide
D. Maintenance of lower filling pressure with pulmonary capillary wedge pressure (PCWP) less
than 8 mm Hg
E. Lung protective ventilation with low tidal volumes of 6 mL/kg of ideal body weight
7. Abdominal compartment syndrome (ACS) is best described by which of the following?
A. Intra-abdominal pressure greater than 15 mm Hg
B. Intra-abdominal pressure greater than 25 mm Hg and evidence of decreased end organ
perfusion (i.e., oliguria, renal dysfunction, hypotension)
C. Intra-abdominal pressure greater than 35 mm Hg with hypoxemia
D. Peak airway pressure greater than 40 cm H2O
8. Which of the following have been shown to be clinical advantages of enteral feeding
versus total parenteral nutrition (TPN) in critically ill surgical patients?
A. Preservation of gut mucosal integrity and barrier function
B. Secretory IgA production of the gut
C. Decreased rates of catheter-related bloodstream infections
D. Lower cost
E. All of the above
9. Which of the following are strict indications to guide the institution of renal replacement
therapy in the form of intermittent hemodialysis or continuous venovenous filtration or
hemodialysis in critically ill surgical patients?
A. Increasing oxygen requirement and chest x-ray findings of interstitial edema and engorged
pulmonary vasculature
B. Blood pH less than 7.25
C. Potassium level greater than 6.0 mEq/L
D. There are no specific or strict indications to start renal replacement therapy
E. CO2 level less than 16 mEq/L on chemistry profile
10. Which of the following is most appropriate regarding glycemic control in critically ill
surgical patients?
A. Goal glucose should be 80 to 110 mg/dL to improve outcomes in patients with traumatic brain
injury.
B. Stress-related hyperglycemia should be managed with longer acting forms of insulin such as
insulin glargine.
C. Maintaining glucose levels less than 180 mg/dL compared with maintaining a range of 81 to
110 mg/dL results in fewer episodes of hypoglycemia and lower mortality.
Chapter 27: Liver Transplantation
1.The most common indication for liver transplantation in the United States is:
A. EtOH
B. Sclerosing cholangitis
C. Hepatitis B virus
D. Hepatitis C virus
E. Hepatocellular carcinoma
2.An infant with extrahepatic biliary atresia, chronic liver insufficiency, and failure to thrive may be
served by:
A. Whole pediatric liver transplantation
B. Split orthotopic liver transplantation
C. Live donor liver transplantation
D. All of the above
E. None of the above
3.The current risk of death to the donor for live donor liver transplantation is:
A. About the same as the risk to a potential kidney donor
B. 1/100
C. 1/1,000
D. 1/10,000
E. 1/100,000
4.The current system of liver distribution is primarily based on:
A. Insurance
B. Medical necessity
C. Region
D. Recipient age
E. None of the above
5.The liver can be divided into segments based on
A. Portal vein inflow
B. Hepatic artery inflow
C. Biliary outflow
D. Hepatic vein outflow
E. All of the above
6.Which are appropriate treatment(s) for hepatocellular carcinoma and cirrhosis?
A.Whole liver transplantation
B. Liver resection
C. Live donor liver transplantation
D. Resection with salvage transplantation
E. All of the above
7.Which statement about immunosuppression medication is false?
A. Cellcept (mycophenolate) is an antiproliferation agent.
B. Calcineurin inhibitors have nephrotoxic and neurotoxic side effects.
C.Calcineurin inhibitors prevent expansion of the host response by inhibiting IL-10 production.
D. Immunosuppressive agents are optionally used in combination to maximize their effect and minimize
toxicity.
Chapter 28: Kidney and Pancreas Transplantation
1.A 50-year-old man arrives in the recovery room following an uneventful living donor kidney
transplantation. It was documented that there was good urine output in the operating room. When the
patient arrives, there is no urine output. The next best step would be to:
A. Order an ultrasound.
B. Perform an emergent biopsy to rule out hyperacute rejection.
C. Examine the patient.
D. Flush the Foley catheter.
2.A 50-year-old man arrives in the recovery room following an uneventful living donor kidney
transplantation. It was documented that there was good urine output in the operating room. When the
patient arrives, there is no urine output. The physical exam reveals BP, 134/70 mm Hg, HR, 76
beats/min, RR, 14 breaths/min, O2 saturation, 100%, Foley is intact. An inspection of the abdomen
reveals a distended right lower quadrant. The incision is dry. The next best step is to:
A. Order an ultrasound.
B. Open the wound at the bedside.
C. Flush the Foley catheter.
D. Return to the operating room.
3.A 50-year-old man arrives in the recovery room following an uneventful living donor kidney
transplantation. It was documented that there was good urine output in the operating room. When the
patient arrives, there is no urine output. The most likely diagnosis in this case is:
A. Lymphocele
B. Urine leak
C. Hyperacute rejection
D. Wound dehiscence
E. Arterial thrombosis
4.A 35-year-old woman with type 1 diabetes wishes to be evaluated for a kidney pancreas
transplantation. She has a history of hypertension and has recently started dialysis. There is no family
history of cardiac disease.
Her initial workup should include:
A. C-peptide level
B. Cardiac stress test
C. Pap smear
D. All of the above
5.A 35-year-old woman with type 1 diabetes wishes to be evaluated for a kidney pancreas
transplantation. She has a history of hypertension and has recently started dialysis. There is no family
history of cardiac disease. The patient presents at 3 weeks post-transplantation with an elevated amylase
level and her creatinine level is also somewhat elevated at 1.4 mg/dL, from a baseline of 1.0 mg/dL. Her
glucose level is normal and her urine output has been normal. Diagnostic tests should include:
A. Abdominal and pelvic CT scan
B. Urinalysis
C. CMV polymerase chain reaction assay
D. Ultrasound and kidney biopsy
E. C-peptide level
Chapter 29: Small Bowel Transplantation
1. Which of the following patients would be considered an appropriate candidate for intestinal
transplantation?
A. A 2-year-old boy with long-segment Hirschsprung disease extending into the jejunum with an end
jejunostomy. He receives a combination of continuous tube feeds via a gastrostomy tube and supplemental
intravenous fluids via a tunneled central venous catheter. He has had the central venous line replaced once
because of a break in the line but has not had a known central venous line infection.
B. A 23-year-old woman with a history of Crohn disease who has had multiple bowel resections because of
strictures. She is currently dependent on TPN, although she can tolerate small amounts of oral intake. Her liver
function is normal. She has had one central venous line infection owing to Staphylococcus epidermidis, which
was successfully treated with antibiotics.
C. A 1-year-old girl who was born prematurely and lost most of her small intestine secondary to necrotizing
enterocolitis. She is currently dependent on TPN and has been hospitalized five times for central venous line
infections, two of which were due to Candida albicans.
D. A 10-year-old boy with pseudo-obstruction who has recurrent episodes of abdominal pain and distention.
He has recently had to begin TPN because of failure to maintain adequate growth.
2. A 2-year-old boy with intestinal failure secondary to gastroschisis and parenteral nutrition–associated
liver disease (PNALD) is on the waiting list for a combined liver-intestine transplant. His blood type is
A, and he weighs 13 kg. Which of the following potential donors would be most appropriate?
A. A 1-year-old donor who sustained head traumas as a result of child abuse; the donor is blood type A and
weighs 8 kg
B. An 8-year-old donor who sustained head trauma as a result of a bicycle versus motor vehicle accident; the
donor is blood type A and weighs 25 kg
C. A 20-year-old donor who sustained head trauma as a result of a motorcycle accident; the donor is blood
type A and weighs 60 kg
D. A 2{1/2}-year-old donor who sustained head trauma as a result of a motor vehicle accident; the donor is
blood type A and weighs 14 kg
3.For patients who require a liver transplant in addition to an intestinal transplant owing to intestinal
failure and parenteral nutrition–associated liver disease (PNALD), what is the advantage of including
the pancreas en bloc with the other organs?
A. Most patients with intestinal failure and PNALD also have diabetes.
B. Including the pancreas avoids having to perform any hilar dissection in the donor organs and avoids having
to perform separate vascular anastomoses for the liver and the intestine during implantation in the recipient.
C. The native pancreas is removed from the recipient with the recipient's liver and remnant small intestine and
needs to be replaced.
D. Including the pancreas has been shown to decrease the incidence of rejection.
4. Which of the following is the most effective induction agent for intestinal transplantation?
A. Alemtuzumab (Campath)
B. Basiliximab (Simulect)
C. Daclizumab (Zenapax)
D. Rabbit antithymocyte globulin (Thymoglobulin)
E. No agent has been proven superior to the others
5.The most effective method of monitoring an intestinal allograft for rejection is:
A. Serum liver function tests
B. Endoscopically obtained mucosal biopsy specimens reviewed by a pathologist
C. Measuring stoma or stool output
D. Clinical signs such as abdominal pain and distention
6.Infection with which of the following pathogens may mimic rejection in intestinal transplant
recipients?
A. Epstein-Barr virus
B. Escherichia coli
C. Cytomegalovirus
D. Enterobacter species
E. Klebsiella species
7.Recipients of which transplanted organ are at the highest risk of developing post-transplant
lymphoproliferative disorder (PTLD)?
A. Kidney
B. Intestine
C. Heart
D. Liver
8.The most common cause of death after intestinal transplantation is:
A. Infection
B. Post-transplant lymphoproliferative disorder (PTLD)
C. Graft-versus-host disease (GVHD)
D. Chronic rejection
Chapter 30: Tumor Biology and Tumor Markers
1.Which of the following genetic changes may be involved in tumorigenesis?
A. Activation of a proto-oncogene
B. Loss of a tumor suppressor gene
C. Activation of a growth factor receptor–encoding gene
D. All of the above
2.Which of the following statements is incorrect?
A. Cancer is the second most common cause of death in the United States.
B. Cancer disproportionally affects people 65 years and older.
C. It is estimated that 15% to 50% of all cancer deaths in the United States can be attributed to overweight and
obesity.
D. Cancer incidence is the number of cancer patients in the population.
3.Which type of adjuvant therapy may provide the best strategy for the postoperative eradication of
residual microscopic disease?
A. Chemotherapy
B. Radiation therapy
C. Immunotherapy
D. Combination therapy
4.Germline mutations have been postulated to be associated with several tumors. Which of the following
is an example?
A. p53
B. APC
C. KRAS
D. All of the above
5.Tumor growth is dependent on:
A. Paracrine factors
B. Autocrine and paracrine factors
C. Immune cell infiltrate
D. Autocrine factors
6.Which early events in the primary tumor are characteristic for the formation of metastases?
A. Angiogenesis
B. Intravasation and extravasation
C. Evasion of cell death
D. Detachment from extracellular matrix and intravasation.
7. What is most essential for the development of a tumor?
A. Successive genetic alterations
B. The ability to produce growth factors
C. Deletion of p53
D. Immunosuppressed or immunodeficient host
8.An ideal tumor marker is
A. Detectable early with a high degree of false-negative findings
B. Detectable only when tumors metastasize
C. Characterized by a high specificity and low sensitivity
D. exclusively by the particular tumor
9.CA 19-9 levels may be elevated in which of the following conditions?
A. Pancreatic adenocarcinoma
B. Benign biliary stricture
C. Malignant biliary stricture
D. Colon cancer
E. All of the above
10.A false-positive CEA test can occur in which of the following benign conditions?
A. Ulcerative colitis
B. Cirrhosis
C. COPD
D. Gallstone pancreatitis
E. All of the above
11.Which of the following is not correct regarding the use of alpha-fetoprotein (AFP) as a tumor marker
for hepatocellular carcinoma (HCC)?
A. AFP levels may be elevated in other gastrointestinal malignancies.
B. The combination of ultrasound with AFP improves the sensitivity of screening for HCC.
C. Following complete resection, AFP levels should fall below 10 ng/mL.
D. The rate at which the AFP level rises is not associated with a worse prognosis.
E. None of the above.
12.Which of the following is correct regarding prostate-specific antigen (PSA)?
A. PSA levels do not rise after digital rectal examination.
B. PSA level > 4 ng/mL is diagnostic of prostate cancer.
C. After complete resection, the PSA level should normalize after 2 to 3 weeks.
D. An elevated but stable PSA level after radiotherapy does not portend clinical relapse.
13..A highly specific test for a tumor marker may still yield a large number of false-positive test results:
A. When the test has low sensitivity
B. When the prevalence of the disease tested for is low in the tested population
C. When the test is done with urine samples instead of serum samples
D. When the assay is a radioimmunoassay
E. When the test has to be done on fresh tumor samples instead of preserved samples
14..Breast cancer specimens are now routinely tested for which of the following tumor markers?
A. Estrogen receptor
B. Progesterone receptor
C. Her2/neu expression
D. A and B
E. A, B, and C
15.Her2/neu expression status of a breast tumor is important for:
A. Monitoring the efficacy of therapy
B. Determining treatment for recurrent cancer
C. Diagnosis
D. Timing of second-look procedures
E. All of the above
16.Patients with metastatic colorectal cancer who lack a KRAS mutation in codon 12 or 1:
A. Are more likely to respond to anti-EGFR antibody therapy
B. Are less likely to respond to anti-EGFR antibody therapy
C. Have improved disease-free survival when treated with anti-EGFR antibody therapy
D. Have no change in overall survival when treated with anti-EGFR antibody therapy
E. A and C
F. A and D
17.Which of the following statements is incorrect regarding the use of the 21-gene assay, Oncotype DX,
in breast cancer?
A. Predicts likelihood of local tumor recurrence
B. Was designed for patients with node-negative, tamoxifen-treated breast cancer
C. Cannot be used in ER-negative tumors
D. Alters treatment choice in approximately 25% of cases
E. Assays 16 tumor-associated gene
Chapter 32: Melanoma and Cutaneous Malignancies
1.What percentage of patients with BCC or SCC develop a second skin cancer within 5 years of the first
skin cancer?
A. 10%
B. 25%
C. 50%
D. 80%
E. 100%
2.After 10 years of immunosuppression, what percentage of transplantation patients develop
malignancies?
A. 1%
B. 10%
C. 25%
D. 50%
E. 80%
3.Which form of skin cancer is associated with the highest risk of simultaneous internal malignancies?
A. BCC
B. SCC
C. Melanoma
D. Extramammary Paget's disease
E. Bowen's disease
4.Which of the following skin malignancies causes the highest number of deaths each year?
A. Squamous cell carcinoma
B. Basal cell carcinoma
C. Merkel cell carcinoma
D. Melanoma
Chapter 34: Bone Tumors
1.Factors that limit local recurrence in low-grade extremity soft tissue sarcoma include all of the
following except:
A.Complete local resection
B. Histologically negative margins
C. Adjuvant external beam radiation therapy
D. Adjuvant brachytherapy
E. Primary presentation
2.Risk factors for local recurrence in extremity soft tissue sarcoma include all of the following except:
A. Positive microscopic margin
B. Fibrosarcoma histopathology
C. Deep location
D. High histologic grade
E. Previous recurrence
3.Risk factors for distant metastasis in extremity soft tissue sarcoma include all of the following except:
A. Recurrent presentation
B. Size = 10 cm
C. Deep location
D. Fibrosarcoma histopathology
E. High histologic grade
4.True statements about soft tissue sarcomas include the following:
A. Approximately 50% occur in the extremities.
B. Prior radiation therapy is a causative agent.
C. Lymphedema is a predisposing factor.
D. Liposarcoma is the most common histopathology
E. All of the above
5.Which one of the following is an important factor indicating poor prognosis in extremity soft tissue
sarcoma?
A. High histologic grade
B. Liposarcoma histopathology
C. Size < 8 cm
D. Distal limb site
E. Prior incisional biopsy
6.Biopsy of a primary bone tumor should be done:
A. As soon as the lesion is discovered
B. After thorough staging studies are completed
C. In the operating room
D. At the same time that a venous access port is placed
7.Which of the following bone tumors are radiographic diagnoses and do not require biopsy?
A. Chondrosarcoma
B. Metastasis
C. Giant cell tumor
D. Osteochondroma
8.Impending fractures should be treated by:
A. Radiation therapy
B. Chemotherapy
C. Internal fixation
D. Bisphosphonates
9.Preoperative chemotherapy for osteogenic sarcoma is:
A. Needed to perform limb-preserving surgery
B. Predictive of disease-free survival
C. Useful to tailor postoperative chemotherapy
D. Determined by the translocation type causing the tumor
10.Joint replacement to reconstruct defects after a tumor resection is:
A. As successful as after removal of an arthritic joint
B. Not possible in children younger than 10 years old
C. Best for benign tumor cases
D. Can be combined with allograft bone transplantation
Chapter 35: Head and Neck
1.Which of the following statements regarding aerodigestive tract cancer associated with human
papillomavirus (HPV) is false?
A. The incidence of HPV-associated aerodigestive tract cancer specific to the tonsil and tongue base subsites is
increasing in North America.
B. HPV-related aerodigestive tract cancer tends to occur in older patients compared with non—HPV-related
aerodigestive tract cancers.
C. The incidence of HPV-related aerodigestive tract cancers is increasing at a higher rate in nonsmokers
compared with smokers.
D. Theincidence of HPV-related aerodigestive tract cancers is increasing at a higher rate in non—alcohol
abusers compared with alcohol abusers.
2.A neck dissection that resects nodal levels I through V but preserves the sternocleidomastoid muscle,
the spinal accessory nerve, and the internal jugular vein is referred to as a:
A. Selective neck dissection
B. Modified neck dissection
C. Radical neck dissection
D. Functional neck dissection
3.To reduce the chance of a vocal fold paralysis postoperatively, the preferred approach to the cervical
spine should be:
A. Left-sided because it reduces the tension on the recurrent laryngeal nerve during the exposure
B. Left-sided because of the incidence of nonrecurrent recurrent laryngeal nerve on the right side
C. Right-sided because it reduces the tension on the recurrent laryngeal nerve during the exposure
D. Right-sided because of the incidence of nonrecurrent recurrent laryngeal nerve on the left side
4.Why should percutaneous tracheotomy performed in the ICU be avoided in patients with prolonged
transoral intubation secondary to ventilator dependence?
A. Percutaneous tracheotomy has a higher incidence of postdecannulation stenosis in patients undergoing
tracheotomy owing to failure to wean from mechanical ventilation.
B. Percutaneous tracheotomy has a higher incidence of complications from accidental decannulation because
an inferiorly based trachea-to-skin flap specific to this procedure is not created.
C. Percutaneous tracheotomy has a higher incidence of "false lumen" creation than traditional open
tracheotomy performed in the operating room.
D. Percutaneous tracheotomy does not have a higher incidence of intraoperative or postoperative
complications and is as safe to perform in ventilator-dependent patients as traditional open tracheotomy
5.An adult patient presents with an asymptomatic, solitary 4-cm neck mass that has been present for 1
month. Physical examination and history are otherwise unremarkable. The initial step in the workup for
this mass would be:
A. CT scan with intravenous contrast agent
B. Open incisional biopsy
C. Fine-needle aspiration
D. Open excisional biopsy with conversion to neck dissection depending on intraoperative frozen section
diagnosis
6.A patient presents with a glottic squamous cell carcinoma involving both anterior true vocal cords but
not either of the arytenoid cartilages, and is not a candidate for endoscopic surgical treatment because
of the inability to expose the larynx for laser excision. A viable conservation surgical therapy would be:
A. Supraglottic laryngectomy
B. Supracricoid laryngectomy with cricohyoidoepiglottopexy
C. Vertical partial laryngectomy
D. Total laryngectomy with tracheoesophageal puncture
7. A 30-year-old woman presents with a slowly growing, asymptomatic, 3-cm parotid mass. Office fine-
needle aspiration is inconclusive. Assuming that the intraoperative frozen section shows this mass to be a
benign salivary neoplasm, the most appropriate surgical plan would be to perform:
A. Incisional biopsy without the need for further surgery
B. Total parotidectomy with facial nerve dissection and preservation
C. Excisional biopsy without the need for further surgery
D. Superficial parotidectomy with facial nerve dissection and preservation
8.A patient has a recurrent laryngeal nerve resection as part of removal of an aggressive thyroid
malignancy. Postoperatively, her voice is breathy and weak and is insufficient for performing at her
profession. The goal of subsequent voice restoration surgery is to:
A. Create volitional abduction and adduction of the affected vocal cord to preserve both voice and airway
patency
B. Create volitional abduction and adduction of the affected vocal cord to preserve both voice and airway
protection from aspiration during swallowing
C. Medialize the affected vocal cord to create static contact with the opposite, mobile cord
D. Place a Silastic implant that is capable of directly contacting the opposite, mobile cord
9.An adult patient presents with a right-sided, 3-cm asymptomatic neck mass deep to the
sternocleidomastoid muscle at the level of the hyoid bone that has been growing over the past 3 months.
The most likely etiology in this patient is:
A. Branchial cleft cyst
B. Malignant lymphadenopathy
C. Lipoma
D. Carotid body tumor
10.The level of the neck nodes bounded by the laryngeal strap muscles anteriorly, the posterior border
of the sternocleidomastoid muscle posteriorly, a horizontal plane at the level of the cricoid cartilage
inferiorly, and the level of the hyoid bone superiorly is referred to as level:
A. Ia
B. II
C. III
D. IV
Chapter 37: Breast Reconstruction
1.A 43-year-old woman is scheduled for reconstruction of the right breast with a latissimus dorsi flap.
She is at greatest risk for which of the following complications?
A. Chronic chest wall pain
B. Dorsal wound dehiscence
C. Partial flap necrosis
D. Seroma
E. Stiffness in the ipsilateral shoulder
2.A 58-year-old nulligravid woman who is scheduled to undergo bilateral prophylactic mastectomy
comes to the office for consultation regarding immediate breast reconstruction. She works full time as a
fitness instructor. The patient currently wears a size 34B brassiere and wants her bra size to be
increased to a C cup, but she wants to make sure that scarring is minimized. Her height is 5 feet, 5
inches and weight is 120 lb. Physical examination of the abdomen shows a paucity of extra tissue. Which
of the following is the most appropriate breast reconstruction procedure for this patient?
A. Bilateral autogenous reconstruction
B. Bilateral tissue expansion followed by implantation of prostheses
C. Delayed breast reconstruction after pathology is confirmed
D. Single-stage reconstruction with prostheses
E. TRAM flap followed by implantation of prostheses
3.A 55-year old woman undergoes a modified radical mastectomy with immediate first-stage
reconstruction of the right breast with a tissue expander. Before beginning the second stage, to exchange
the tissue expander with a permanent prosthesis, pathology results from analysis of tissue from the right
breast indicate metastatic carcinoma of four axillary lymph nodes. Radiation therapy is recommended.
Which of the following interventions will result in the best long-term appearance of the reconstructed
breast?
A. Complete the tissue expansion before radiation and exchange the tissue expander with a prosthesis after
radiation.
B. Deflate the tissue expander before radiation; reinflate the tissue expander and exchange with a prosthesis
after radiation.
C. Remove the tissue expander and reconstruct the breast with a TRAM flap before radiation.
D. Remove the tissue expander before radiation; after radiation, reinsert and expand a tissue expander and then
exchange with a prosthesis.
E. Remove the tissue expander before radiation and reconstruct the breast with a TRAM flap after radiation
4.The skin-sparing mastectomy involves the preservation of the:
A. Areola
B. Nipple
C. Inframammary crease
D. Skin
E. Skin and areola
5.A 45-year-old woman with T3N0 invasive ductal carcinoma in the inferior pole of the left breast is
scheduled to undergo segmental mastectomy and subsequent radiation therapy. She currently wears a
size 36DDD bra and is willing to accept any cup size from C to DDD. Which of the following
interventions will yield the best cosmetic result in this patient?
A. Bilateral reduction mammaplasty
B. Implantation of a prosthesis in the left breast and mastopexy of the right breast
C. Latissimus dorsi musculocutaneous flap reconstruction of the left breast and mastopexy of the right breast
D. TRAM reconstruction of the defect
E. No reconstruction
6.A 40-year-old woman is scheduled to undergo reconstruction of the right breast via a free TRAM flap.
She has smoked two packs of cigarettes daily for the past 8 years. This patient's smoking history
increases her risk of which of the following postoperative complications?
A. Hematoma
B. Mastectomy flap necrosis
C. Seroma
D. TRAM flap loss
E. Vessel thrombosis
7.A 48-year-old woman comes to the office for consultation regarding reconstruction of her right breast
after mastectomy because of cancer. The patient is concerned about maximizing the aesthetic result and
minimizing any donor site deformity. Physical examination shows a well-healed chest wall and a B cup
left breast with grade 3 ptosis. Soft tissue reconstruction with an SGAP free flap is planned. Which of
the following is a disadvantage of this procedure?
A. Difficulty molding the gluteal fat
B. Gait dysfunction
C. Inability to hide the donor scar completely
D. Inability to provide a sensate flap
E. Lack of abundant soft tissue
8.A healthy 27-year-old woman comes to the office for follow-up examination 3 months after undergoing
bilateral prophylactic mastectomy and reconstruction with saline prostheses. She is concerned about the
incision on her left breast because it is slightly swollen and warm to the touch. Physical examination
shows mild erythema of the left breast and normal healing of the right breast. Which of the following is
the most appropriate initial management?
A. Observation
B. Oral antibiotic therapy
C. Open lavage of the implant pocket
D. Removal of the prosthesis
E. Exchange of the prosthesis
9.A 36-year-old woman comes to the office for consultation regarding breast reconstruction 1 year after
undergoing a right modified radical mastectomy. The procedure was followed by 6 weeks of radiation
therapy. She has no history of other surgical procedures or serious medical illnesses. She is 5 feet. 4
inches in height and weighs 135 lb. She wears a size 32B brassiere. Which of the following is the least
appropriate breast reconstruction procedure for this patient?
A. Extended latissimus dorsi flap
B. Latissimus dorsi flap with saline-filled prosthesis
C. SGAP flap
D. TRAM flap
E. Two-stage reconstruction with a tissue expander and saline-filled prosthesis
10.Secondary breast reconstruction procedures such as nipple reconstruction and areolar tattooing are
considered:
A. At time of mastectomy and immediate breast reconstruction
B. Prior to the patient receiving chemotherapy
C. Prior to the patient receiving radiotherapy
D. After chemotherapy and radiation therapy
E. Prior to flap or expander placement
Chapter 38: Thyroid
1.The recurrent laryngeal nerve provides motor innervation to all muscles of the larynx.
A. The recurrent laryngeal nerve provides motor innervation to all muscles of the larynx.
B. The thyroid forms as two lateral projections from the developing alimentary tract that fuse in the midline
during development.
C. Unilateral vocal cord paralysis may have minimal clinical symptoms.
D. All cells in the adult thyroid are of endodermal origin.
2.You evaluate a 35-year-old woman with a palpable nodule in the right lateral neck. No other lateral
neck masses are appreciable on examination. Fine-needle aspiration biopsy of this mass reveals thyroid
cells. Which of the following statements is true?
A. In the absence of a palpable thyroid nodule, this likely represents lateral aberrant thyroid, which is a normal
embryologic variant.
B. When found in the setting of the most common pathologic type of thyroid cancer, this finding does not
significantly alter the expected survival for this patient.
C. Given the pathologic finding, the appropriate surgical approach is total thyroidectomy and selective
excision of all lymph nodes that appear suspicious on operative evaluation.
D. In the absence of palpable adenopathy, it would be unlikely to find pathologic evidence of metastasis to
regional lymph nodes from the most common type of thyroid cancer.
3.You evaluate a 27-year-old woman with a thyroid nodule. Fine-needle aspiration biopsy is consistent
with medullary thyroid cancer. Which of the following statements is true?
A. Most cases of medullary thyroid cancer are sporadic and not associated with a familial syndrome.
B. The next appropriate step in management is to proceed directly to total thyroidectomy with bilateral central
compartment lymph node dissection.
C. In a case of the finding of RET proto-oncogene mutation in a patient with no palpable or ultrasound-
detectable thyroid nodule, annual observation is indicated, and thyroidectomy should be performed when a
nodule is first appreciated on examination or imaging.
D. After resection for medullary thyroid cancer and recovery from surgery, radioiodine ablation with either
thyroid hormone withdrawal or thyroid-stimulating hormone (TSH) administration is the next therapeutic
intervention.
4.All of the following statements regarding the treatment of papillary thyroid cancer are true except
which one (which is false)?
A. Total thyroidectomy is indicated for cancers greater than 1 cm in greatest dimension and for smaller cancers
with adverse features such as a history of radiation exposure.
B. Reoperation is indicated for completion thyroidectomy for patients who have a papillary thyroid cancer
greater than 1 cm discovered on permanent section pathology after thyroid lobectomy.
C. Prophylactic lateral compartment neck dissection (ipsilateral modified radical neck dissection) is indicated
in most cases.
D. After resection, exogenous thyroid hormone should be titrated to achieve a subnormal thyroid-stimulating
hormone (TSH).
5.All of the following statements are characteristic of hormones produced by the thyroid gland except
which one (which is false)?
A. Iodine is essential for the production of thyroid hormones.
B. The enzyme thyroid peroxidase is an integral part of thyroid hormone production.
C. Thyroglobulin (Tg) is the storage form of thyroid hormone.
D. Thyroid hormone production and release are predominantly regulated by thyroid-stimulating hormone
(TSH) from the pituitary gland.
E. Most hormone released by the thyroid is triiodothyronine (T3).
6.Which of the following statements is true?
A. Thyroglobulin (Tg) has predictive value for the recurrence of well-differentiated thyroid cancer.
B. After surgery, calcitonin is replaced by a once-daily oral regimen to maintain calcium homeostasis.
C. Thyroid-stimulating hormone (TSH) levels should be maintained in the high to normal range after surgery
for well-differentiated thyroid cancer.
D. Corticosteroids stimulate thyroid hormone release and enhance peripheral conversion of thyroxine (T4) to
triiodothyronine (T3).
E. All statements are true.
7.Complete surgical resection (thyroidectomy) is first-line therapy for which one of the following?
A. Riedel thyroiditis (struma)
B. Toxic nodular goiter with marked compressive symptoms
C. Acute suppurative thyroiditis
D. Uncomplicated Graves disease (diffuse toxic goiter)
E. Hashimoto thyroiditis
8.You are preparing a patient for total thyroidectomy for treatment of Graves disease (diffuse toxic
goiter). To attempt to avoid complications from severe thyrotoxicosis, including life-threatening thyroid
storm, you could employ any of the following therapies except which one (which is not appropriate)?
A. Beta blockade with an agent such as propranolol
B. Large doses of thionamides such as propylthiouracil (PTU) and methimazole
C. Large doses of iodine after a thionamide
D. Rapid fluid replacement along with corticosteroids
E. Large doses of amiodarone
9.Which of the following is associated with an aberrant takeoff of the right subclavian artery?
A. Nonrecurrent left laryngeal nerve
B. Absence of the right thyroid lobe
C. Absence of the thyroid isthmus
D. Nonrecurrent right laryngeal nerve
E. Nonrecurrent right external branch of the superior laryngeal nerve
10.A palpable lymph node is found along the posterior-lateral border of the anterior belly of the
digastric muscle. This node lies in which anatomic lymph node basin?
A. IA
B. IB
C. IIA
D. IIB
E. VI
Chapter 39: The Parathyroid Glands
1.Embryologic aspects of the superior parathyroid gland include which of the following?
A. Origin from branchial pouch III
B. Origin from branchial pouch V
C. Typically associated with the thyroidal tubercle of Zuckerkandl
D. Frequent ectopic location within thyroid parenchyma
2.Normally embedded in fat and located within a 2-cm circumscribed area that is cranial to the
intersection of the inferior thyroid artery and recurrent laryngeal nerve is a description that applies to
the:
A. Superior parathyroid gland
B. Inferior parathyroid gland
C. Pyramidal lobe of thyroid gland
D. Thymus
3.Which of the following is true regarding human parathyroid hormone?
A. Intact parathyroid hormone is 92 amino acids long.
B. Parathyroid hormone is principally metabolized in the kidney.
C. Biologically active parathyroid hormone fragments include the C-terminal sequence of the intact molecule.
D. Parathyroid hormone secretion is closely linked with the intracellular concentration of ionized calcium.
4.An adult with a serum calcium concentration of 10.6 mg/dL (normal, 8.6 to 10.2 mg/dL), serum intact
PTH level of 90 pg/mL (normal, 10 to 65 pg/mL), and a 24-hour urinary calcium value of 25 mg/day has:
A. Secondary hyperparathyroidism
B. Tertiary hyperparathyroidism
C. A defect in the calcium-sensing receptor gene
D. A syndrome successfully treated with surgery
5.Parathyroid carcinoma:
A. Is most common at the extremes of age
B. Is often associated with mild hypercalcemia
C. Is optimally treated with en bloc resection of the ipsilateral thyroid lobe at the initial operation
D. Is always easy to recognize at the time of operation
6.Patients with asymptomatic primary hyperparathyroidism should undergo parathyroidectomy when:
A. The serum calcium concentration is 1 mg/dL above the upper limit of normal.
B. They are unwilling to undergo medical surveillance.
C. They are younger than 50 years.
D. The T-score is less than –2.5 on bone mineral density measurement.
E. All of the above
7.Ectopic locations for superior parathyroid adenomas include:
A. In the tracheoesophageal groove near the esophagus
B. In a retroesophageal plane in the upper posterior mediastinum
C. Undescended near the submandibular gland
D. All of the above
8.Which of the following is correct about intraoperative parathyroid hormone monitoring?
A. Quick intraoperative chemiluminescent assays measure only the N-terminal fragment of PTH.
B. The most commonly used criterion for cure is a 25% decrease from baseline at 10 minutes postexcision.
C. Monitoring is practical because intact PTH has a very short half-life.
D. The value is never affected by aggressive dissection of the adenoma.
9.Common reasons for persistent hyperparathyroidism after initial parathyroidectomy include:
A. Failure to find the causative adenoma at the initial operation
B. Failure to detect or recognize a second adenoma (double adenoma)
C. Failure to recognize and aggressively treat multigland hyperplasia
D. All of the above
10.Parathyroidectomy for secondary hyperparathyroidism is indicated when:
A. Patients have refractory bone or joint pain and muscular weakness.
B. Calciphylaxis occurs.
C. Tertiary hyperparathyroidism is evident.
D. All of the above
Chapter 40: Endocrine Pancreas
1. The endocrine tumor of the pancreas with the lowest rate of malignancy is:
A. Glucagonoma
B. Somatostatinoma
C. Gastrinoma
D. Insulinoma
2. Which of the following actions is not performed by insulin?
A. Decreasing blood sugar level
B. Decreasing protein synthesis
C. Decreasing glycogenolysis
D. Decreasing lipolysis
E. Increasing glucose transport
3. The best means to localize a gastrinoma preoperatively is:
A. Enhanced MRI
B. Selective portal venous sampling
C. Calcium angiography
D. Somatostatin receptor scintigraphy
E. Endoscopic ultrasound
4. In patients with severe diarrhea, continuous nasogastric aspiration may provide a
diagnostic clue for:
A. Sprue
B. Zollinger-Ellison syndrome
C. Cholera
D. Verner-Morrison (VIPoma) syndrome
E. Glucagonoma
5. Which of the following endocrine tumors of the pancreas has the best prognosis for cure?
A. Insulinoma
B. Somatostatinoma
C. VIPoma
D. Glucagonoma
E. Gastrinoma
6. The most effective intraoperative technique for localizing a pancreatic gastrinoma
involves palpation plus:
A. Intraoperative endoscopy with transillumination
B. Selective venous sampling
C. Intra-arterial injection of vital blue dye
D. Intraoperative ultrasonography
E. Caudal pancreatectomy
7. MEN1 syndrome is usually associated with tumors in all but which of the following
glands?
A. Thyroid
B. Adrenal
C. Pituitary
D. Pancreas
E. Parathyroid
8. The most accurate method for localizing an insulinoma before operation is:
A.MRI
B. CT
C. Somatostatin receptor scintigraphy
D. Selective angiography
E. Intra-arterial calcium stimulation
9. Clinical clues to Zollinger-Ellison syndrome include all the following except:
A. Hypercalcemia
B. Gastroesophageal reflux disease (GERD)
C. A positive corticotropin test
D. Diarrhea
E. High output of gastric acid
10. The most common site for gastrinomas is the:
A. Head of pancreas in gastrinoma triangle
B. Prepyloric area
C. Body and tail of pancreas
D. Duodenum
E. Gastroduodenal ligament
11. The salient clinical characteristic of the glucagonoma syndrome is:
A. Brittle hyperglycemia
B. High serum calcium
C. Necrolytic migrating erythema
D. Secretory diarrhea
E. Massive fatty infiltration of the liver
Chapter 43: Esophagus
1. Which of the following statements about esophageal anatomy is correct?
A. The esophagus has a poor blood supply, which is segmental in distribution and accounts for
the high incidence of anastomotic leakage.
B. The esophageal serosa consists of a thin layer of fibroareolar tissue.
C. The esophagus has two distinct muscle layers—an outer, longitudinal and an inner, circular
layer—that are striated in the upper third and smooth in the distal two thirds.
D. Injury to the recurrent laryngeal nerve results in vocal cord dysfunction, but does not affect
swallowing.
E. The lymphatic drainage of the esophagus is sparse, localized primarily to adjacent
paraesophageal lymph nodes.
2. Which of the following statements about achalasia is correct?
A. In most cases in North America, the cause is a parasitic infestation by Trypanosoma cruzi.
B. Chest pain and reflux are the usual symptoms.
C. Distal third esophageal adenocarcinomas may occur in 30% of patients within 5 years of
diagnosis.
D. Manometry shows failure of LES relaxation on swallowing and absent or weak simultaneous
contractions in the esophageal body after swallowing.
E. Endoscopic botulinum toxin injection of the LES, pneumatic dilation, and esophagomyotomy
can provide highly effective curative therapy for achalasia.
3. Which of the following statements regarding the pathology of esophageal carcinoma is
correct?
A. Worldwide, adenocarcinoma is the most common esophageal malignancy.
B. Squamous cell carcinoma is most common in the distal esophagus, whereas adenocarcinoma
predominates in the middle third.
C. Patients with Barrett's metaplasia are more likely than the general population to develop
adenocarcinoma.
D. Metastases from esophageal carcinoma are characteristically limited to regional mediastinal l
lymph nodes adjacent to the tumor.
E. Achalasia, radiation esophagitis, caustic esophageal stricture, and Plummer-Vinson syndrome
are premalignant esophageal lesions that predispose to the development of adenocarcinoma.
4. Which of the following statements about the surgical treatment of esophageal carcinoma
is correct?
A. The finding of severe dysplasia in association with Barrett's mucosa is an indication for an
antireflux operation to prevent subsequent development of carcinoma.
B. Long-term survival at all stages is improved by radical en bloc resection of the esophagus
with its contained tumor, adjacent mediastinal tissues, and regional lymph nodes.
C. The morbidity and mortality rates for cervical esophagogastric anastomotic leak are lower
than the rates associated with intrathoracic esophagogastric anastomotic leak.
D. The leading complications of transthoracic esophagectomy and intrathoracic esophagogastric
anastomosis are bleeding and wound infection.
E. Transhiatal esophagectomy without thoracotomy achieves better long-term survival than
transthoracic esophagectomy.
5. The best management for a 48-hour-old distal esophageal perforation is:
A. Antibiotics and drainage
B. Division of the esophagus and exclusion of the perforation
C. Primary repair with buttressing
D. Resection with cervical esophagostomy, gastrostomy, and jejunostomy
E. T-tube fistula and drainage
6. A 42-year-old man with a history of alcoholism and tobacco use is diagnosed by barium
swallow with an esophageal stricture 10 cm from the gastroesophageal junction.
Esophagoscopy confirms a moderate stricture, and biopsy reveals fibrosis. What is the next
step in treatment?
A. Diet alteration and acid suppression with H2 inhibitors
B. Endoscopic stricture dilation and acid supression with high-dose proton pump inhibitor (PPI)
therapy
C. Fluoroscopy-guided stent placement
D. Esophagectomy with gastric pull-through
E. Laparoscopic fundoplication
7. The most common cause of morbidity after esophagectomy is:
A. Postoperative hemorrhage
B. Pneumonia
C. Anastomotic leak
D. Recurrent laryngeal nerve injury
E. Chylothorax
8. A 54-year-old woman has severe chest pain intermittently after meals. An extensive
cardiac workup has been negative. Esophageal manometry before and after swallowing a
5-mL water bolus shows lower esophageal sphincter resting pressure of 30 mm Hg, length
of 3 cm, and residual pressure of 5 mm Hg. Peristaltic contractions have an amplitude of
204 mm Hg 5 cm above the lower esophageal sphincter and a duration of 7 seconds. These
measurements are most consistent with:
A. Nutcracker esophagus
B. Achalasia
C. Diffuse esophageal spasm
D. Nonspecific esophageal dysmotility
E. Hypertensive lower esophageal sphincter
9. The most crucial step in the management of a patient with Zenker's diverticulum is to:
A. Identify the underlying motility disorder preoperatively
B. Perform a complete cricopharyngotomy at operation
C. Resect the diverticulum at operation
D. Drain the surgical site postoperatively
E. Perform an antireflux procedure at the time of surgery
10.Leiomyomas of the esophagus:
A. Require esophagectomy for definitive treatment because of submucosal spread
B. Appear as a distinct rough-edged mass on barium swallow
C. Account for <25% of benign esophageal tumors
D. Do not require biopsy for preoperative diagnosis
E. Metastasize primarily to supraclavicular nodes
Chapter 44: Hiatal Hernia and Gastroesophageal Reflux Disease
1. Components of the lower esophageal sphincter include all of the following except:
A. Sling fibers of the cardia
B. Striated external sphincter muscle fibers
C. Diaphragm
D. Intrinsic muscle of the distal esophagus
E. Transmitted pressure of the abdominal cavity
2. A partial fundoplication (<360 degrees) should always be performed in patients with
ineffective esophageal motility (IEM).
A. True
B. False
3. After antireflux surgery:
A. Short-term bloating occurs in more than 60% of patients
B. Long-term dysphagia is present in 50% of patients
C. Dilation should be performed for any dysphagia in the first 2 months
D. Long-term bloating may be the direct result of vagal nerve injury
4. Which of the following statements concerning extraesophageal manifestations of GERD
is true?
A. In some patients, GERD is manifested only with extraesophageal symptoms (i.e., these
patients have no heartburn).
B. Diagnosing the association of GERD with extraesophageal symptoms is always possible if all
diagnostic tools are used.
C. Extraesophageal symptoms respond to medical and surgical antireflux therapy to the same
degree as heartburn.
D. In patients with GERD, extraesophageal symptoms are caused by vagal-mediated pathways.
5. Paraesophageal hernias:
A. Have a high likelihood of acute strangulation, and all should be repaired when identified even
if asymptomatic
B. Are rarely associated with chronic gastrointestinal blood loss
C. Should be repaired only in patients with symptoms
D. Are best diagnosed by endoscopy
Chapter 45: Abdominal Wall, Umbilicus, Peritoneum, Mesenteries,
Omentum, and Retroperitoneum
1. Which of the following statements is true regarding the anatomy of the abdominal wall?
A. The inguinal or Poupart's ligament is the lower free edge of the aponeurosis of the transversus
abdominis muscle.
B. The rectus sheath is composed of aponeurotic fibers from the internal and external oblique
and transversus abdominis muscles.
C. The rectus sheath completely envelops the anterior and posterior surfaces of the rectus a
abdominis muscle from its origin in the costal cartilages to its insertion on the pubis.
D. The lymphatic vessels of the abdominal wall drain almost exclusively into the superficial and
deep inguinal lymph node chains.
E. The abdominal wall is innervated almost exclusively from branches of the first through fourth
lumbar spinal nerves.
2. By definition, a hernia results from a defect in which of the following structures?
A. Transversalis fascia
B. Peritoneum
C. Internal oblique fascia
D. Internal and external oblique fascia
E. Rectus sheath
3. Which of the following are not contained within the preperitoneal space?
A. The medial umbilical ligaments, which are the remnants of the obliterated umbilical arteries
B. The median umbilical ligament, which is the remnant of the obliterated allantoic stalk (or
urachus)
C. The inferior epigastric arteries and veins
D. The superior epigastric arteries and veins
E. The falciform ligament containing the round ligament of the liver, which is the remnant of the
obliterated umbilical vein
4. All the following are abnormalities related to persistence of the omphalomesenteric duct
except:
A. Meckel's diverticulum
B. Omphalocele
C. Enterocutaneous fistula
D. Umbilical sinus
E. Intestinal obstruction from volvulus
5. All the following factors have been related to the development of rectus sheath
hematomas except:
A. Pregnancy
B. Strenuous muscular exertion
C. Anticoagulation
D. Benign and malignant tumors of the abdominal wall
E. Trauma
6. All the following statements are true regarding the natural history of desmoid tumors of
the abdominal wall except:
A. Patients with familial adenomatous polyposis syndrome have a much higher incidence of
desmoid tumors than does the general population.
B. Tumors manifest as painless enlarging masses, often with a relatively rapid rate of growth.
C. Most patients have systemic metastases to the liver and lung at the time of presentation.
D. The development of abdominal wall desmoid tumors is often temporally related to a recent
childbirth or abdominal operation or trauma.
7. The clinical features of an abdominal wall mass that should suggest a malignant cause
include all the following except:
A. The occurrence of severe unrelenting pain in an abdominal wall mass
B. Nonreducible masses arising from below the superficial fascia
C. Size > 5 cm
D. A recent increase in size
E. Fixation to the abdominal wall or underlying organs
8. The peritoneum and peritoneal cavity respond to infection by all the following except:
A. The rapid clearance of particulate matter, including bacteria, via the normal circulation of
peritoneal fluid toward and ultimately into the diaphragmatic lymphatic channels and thoracic
duct.
B. The generation of potent anti-inflammatory mediators by peritoneal macrophages, which
prevent the movement of leukocytes into the peritoneal cavity from the surrounding
microvasculature
C. The release of histamine and other vasoactive products by degranulating peritoneal mast cells,
causing the extravasation of protein-rich fluid into the peritoneal space
D. The opsonization of bacteria by proteins within the fluid generated by the inflamed peritoneal
Membrane
E. The sequestration of bacteria within fibrin matrices, promoting abscess formation and limiting
the generalized spread of infection throughout the peritoneal cavity
9. Which of the following are important causes of ascites?
A. Cirrhosis
B. Peritoneal carcinomatosis
C. Traumatic injuries to the retroperitoneal lymphatic channels
D. Nephrotic syndrome
E. All of the above
10. All the following are valuable diagnostic measures in the evaluation of cirrhotics with
new-onset ascites except:
A. Physical examination showing a full bulging abdomen, with dullness of the flanks on
percussion
B. Paracentesis with measurement of ascitic fluid cell count and differential
C. Paracentesis with measurement of ascitic fluid albumin concentration
D. Determination of the serum-albumin to ascites-albumin gradient (SAAG)
E. Paracentesis with Gram staining of the ascitic fluid
11. All the following statements are true regarding spontaneous bacterial peritonitis (SBP)
except:
A. SBP occurs almost exclusively in patients with high protein concentrations in the ascitic fluid
(i.e., SAAG < 1.1 g/dL).
B. The most common pathogens in adults are Escherichia coli and Klebsiella pneumoniae,
whereas in children streptococcal species and Staphylococcus aureus play a greater role.
C. The diagnosis of SBP is made initially by showing >250 neutrophils/mm3
of ascitic fluid in
the setting of abdominal pain, fever, or peripheral leukocytosis or some combination of these.
D. Gram staining of ascitic fluid usually does not show bacterascites.
12. All the following statements are true regarding intra-abdominal hernias except:
A. Mesocolic (or paraduodenal) hernias result from abnormalities of embryonic midgut rotation.
B. The superior mesenteric artery and vein course along the medial border of the neck of right
and left mesocolic hernias.
C. Patients with intra-abdominal hernias present most commonly with signs and symptoms of
intestinal obstruction.
D. Internal hernias may result from the creation of mesenteric windows at the time of bowel
resections.
E. All of the above are true.
13. All the following are advantages to a retroperitoneal operative approach compared
with a transabdominal approach to retroperitoneal organs except:
A. Less postoperative ileus with a more rapid resumption of a normal diet
B. Less operative time and blood loss
C. No intra-abdominal adhesions, reducing the likelihood of subsequent small bowel
obstructions
D. Less intraoperative evaporative fluid losses with less dramatic intravascular fluid shifts
E. Fewer respiratory complications, such as atelectasis or pneumonia
Chapter 46: Hernias
1. The best outcome for recurrent inguinal hernia repair is achieved by:
A. Selecting the same approach used for the initial repair
B. Experienced hernia surgeons at hernia centers
C. Cooper ligament repair
D. Using prosthetic mesh
E. Bassini repair
2. Which of the following is not a common cause of inguinal hernia recurrence?
A. Excessive suture line tension
B. Failure to recognize an indirect hernia
C. Increased intra-abdominal pressure
D. Failure to a use tension-free or mesh repair
E. Poor collagen formation
3. Failure to recognize which of the following during repair of a sliding inguinal hernia is
associated with increased complications?
A. The colon usually constitutes part of the hernia sac.
B. Sliding hernias are usually direct hernias.
C. A separate incision is necessary for reduction.
D. Tension-free repair should be avoided.
E. High ligation of the sac is usually necessary.
4. Femoral hernias are:
A. The most common type of hernia in women
B. Usually bilateral
C. Strangulated in 15% to 20% of cases
D. More common on the left side
E. Associated with an inguinal hernia in >50% of patients
5. Nerve injury during laparoscopic hernia repair is best avoided by not placing tacks:
A. Medial to the pubic tubercle
B. Superior to the lateral iliopubic tract
C. Inferior to the lateral iliopubic tract
D. In the Cooper ligament
E. In the peritoneum
03.sabiston surgery questions 17th ed
03.sabiston surgery questions 17th ed
03.sabiston surgery questions 17th ed
03.sabiston surgery questions 17th ed
03.sabiston surgery questions 17th ed
03.sabiston surgery questions 17th ed
03.sabiston surgery questions 17th ed
03.sabiston surgery questions 17th ed
03.sabiston surgery questions 17th ed
03.sabiston surgery questions 17th ed
03.sabiston surgery questions 17th ed
03.sabiston surgery questions 17th ed
03.sabiston surgery questions 17th ed
03.sabiston surgery questions 17th ed
03.sabiston surgery questions 17th ed
03.sabiston surgery questions 17th ed
03.sabiston surgery questions 17th ed
03.sabiston surgery questions 17th ed
03.sabiston surgery questions 17th ed
03.sabiston surgery questions 17th ed
03.sabiston surgery questions 17th ed
03.sabiston surgery questions 17th ed
03.sabiston surgery questions 17th ed
03.sabiston surgery questions 17th ed
03.sabiston surgery questions 17th ed
03.sabiston surgery questions 17th ed
03.sabiston surgery questions 17th ed
03.sabiston surgery questions 17th ed
03.sabiston surgery questions 17th ed
03.sabiston surgery questions 17th ed
03.sabiston surgery questions 17th ed
03.sabiston surgery questions 17th ed
03.sabiston surgery questions 17th ed
03.sabiston surgery questions 17th ed
03.sabiston surgery questions 17th ed
03.sabiston surgery questions 17th ed
03.sabiston surgery questions 17th ed
03.sabiston surgery questions 17th ed
03.sabiston surgery questions 17th ed
03.sabiston surgery questions 17th ed
03.sabiston surgery questions 17th ed
03.sabiston surgery questions 17th ed
03.sabiston surgery questions 17th ed
03.sabiston surgery questions 17th ed
03.sabiston surgery questions 17th ed
03.sabiston surgery questions 17th ed
03.sabiston surgery questions 17th ed
03.sabiston surgery questions 17th ed

More Related Content

What's hot

Liver tumors &amp; liver transplantation
Liver tumors &amp; liver transplantationLiver tumors &amp; liver transplantation
Liver tumors &amp; liver transplantation
surgerymgmcri
 
Abcd of lapchole
Abcd of lapchole     Abcd of lapchole
Choledochal cyst
Choledochal cyst Choledochal cyst
Choledochal cyst
Gagan Adhikari
 
Gastroenterology SCE MCQ
Gastroenterology SCE MCQGastroenterology SCE MCQ
Gastroenterology SCE MCQ
juuraju
 
Familial adenomatous polyposis
Familial adenomatous polyposisFamilial adenomatous polyposis
Familial adenomatous polyposisasteinman
 
Conduits after esophagectomy for esophageal reconstruction
Conduits after esophagectomy for esophageal reconstruction Conduits after esophagectomy for esophageal reconstruction
Conduits after esophagectomy for esophageal reconstruction
Shahbaz Panhwer
 
Pediatric Surgery spectrum
 Pediatric Surgery spectrum Pediatric Surgery spectrum
Pediatric Surgery spectrum
Selvaraj Balasubramani
 
Management of duodenal trauma
Management of duodenal traumaManagement of duodenal trauma
Management of duodenal trauma
Uday Sankar Reddy
 
Bile duct injuries in Laparocsopic cholecystectomy
Bile duct injuries in Laparocsopic cholecystectomyBile duct injuries in Laparocsopic cholecystectomy
Bile duct injuries in Laparocsopic cholecystectomy
UCMS-TH Bhairahwa, NEPAL
 
Git mcqs dr.ahmed_mowafy
Git mcqs dr.ahmed_mowafyGit mcqs dr.ahmed_mowafy
Git mcqs dr.ahmed_mowafy
czer Shmary
 
Gastrointestinal mcq
Gastrointestinal mcqGastrointestinal mcq
Gastrointestinal mcq
Rashed Hassen
 
Bile duct injuries.slideshare
Bile duct injuries.slideshareBile duct injuries.slideshare
Bile duct injuries.slidesharedrksreenath
 
Peritoneal carcinomatosis
Peritoneal carcinomatosisPeritoneal carcinomatosis
Peritoneal carcinomatosis
Dr Dharma ram Poonia
 
Gastric Perforation
Gastric PerforationGastric Perforation
Gastric Perforation
AnneSaputra
 
Internal Medicine Sample Questions
Internal Medicine Sample QuestionsInternal Medicine Sample Questions
Internal Medicine Sample Questions
DJ CrissCross
 
Bile leaks after lapchole Nir Hus MD., PhD.
Bile leaks after lapchole Nir Hus MD., PhD.Bile leaks after lapchole Nir Hus MD., PhD.
Bile leaks after lapchole Nir Hus MD., PhD.
Nir Hus MD, PhD, FACS
 
Lower Gastro-Intestinal Bleed
Lower Gastro-Intestinal BleedLower Gastro-Intestinal Bleed
Lower Gastro-Intestinal BleedAnshuman Aashu
 
Damage control surgery
Damage control surgeryDamage control surgery
Damage control surgery
Bashir BnYunus
 
Quiz gastro
Quiz   gastroQuiz   gastro
Quiz gastro
Rohit Rajeevan
 

What's hot (20)

Liver tumors &amp; liver transplantation
Liver tumors &amp; liver transplantationLiver tumors &amp; liver transplantation
Liver tumors &amp; liver transplantation
 
Abcd of lapchole
Abcd of lapchole     Abcd of lapchole
Abcd of lapchole
 
Choledochal cyst
Choledochal cyst Choledochal cyst
Choledochal cyst
 
Gastroenterology SCE MCQ
Gastroenterology SCE MCQGastroenterology SCE MCQ
Gastroenterology SCE MCQ
 
Familial adenomatous polyposis
Familial adenomatous polyposisFamilial adenomatous polyposis
Familial adenomatous polyposis
 
Surgery Osce Quiz 4
Surgery Osce Quiz 4Surgery Osce Quiz 4
Surgery Osce Quiz 4
 
Conduits after esophagectomy for esophageal reconstruction
Conduits after esophagectomy for esophageal reconstruction Conduits after esophagectomy for esophageal reconstruction
Conduits after esophagectomy for esophageal reconstruction
 
Pediatric Surgery spectrum
 Pediatric Surgery spectrum Pediatric Surgery spectrum
Pediatric Surgery spectrum
 
Management of duodenal trauma
Management of duodenal traumaManagement of duodenal trauma
Management of duodenal trauma
 
Bile duct injuries in Laparocsopic cholecystectomy
Bile duct injuries in Laparocsopic cholecystectomyBile duct injuries in Laparocsopic cholecystectomy
Bile duct injuries in Laparocsopic cholecystectomy
 
Git mcqs dr.ahmed_mowafy
Git mcqs dr.ahmed_mowafyGit mcqs dr.ahmed_mowafy
Git mcqs dr.ahmed_mowafy
 
Gastrointestinal mcq
Gastrointestinal mcqGastrointestinal mcq
Gastrointestinal mcq
 
Bile duct injuries.slideshare
Bile duct injuries.slideshareBile duct injuries.slideshare
Bile duct injuries.slideshare
 
Peritoneal carcinomatosis
Peritoneal carcinomatosisPeritoneal carcinomatosis
Peritoneal carcinomatosis
 
Gastric Perforation
Gastric PerforationGastric Perforation
Gastric Perforation
 
Internal Medicine Sample Questions
Internal Medicine Sample QuestionsInternal Medicine Sample Questions
Internal Medicine Sample Questions
 
Bile leaks after lapchole Nir Hus MD., PhD.
Bile leaks after lapchole Nir Hus MD., PhD.Bile leaks after lapchole Nir Hus MD., PhD.
Bile leaks after lapchole Nir Hus MD., PhD.
 
Lower Gastro-Intestinal Bleed
Lower Gastro-Intestinal BleedLower Gastro-Intestinal Bleed
Lower Gastro-Intestinal Bleed
 
Damage control surgery
Damage control surgeryDamage control surgery
Damage control surgery
 
Quiz gastro
Quiz   gastroQuiz   gastro
Quiz gastro
 

Viewers also liked

Medical student surgery osce
Medical student surgery osceMedical student surgery osce
Medical student surgery osce
Bashir BnYunus
 
500 single best answers in medicine
500 single best answers in medicine500 single best answers in medicine
500 single best answers in medicine
hamadadodo
 
Dentistry mcqs
Dentistry mcqsDentistry mcqs
Dentistry mcqs
Faiz Rehman
 
Mcq 1060 questions
Mcq 1060 questionsMcq 1060 questions
Mcq 1060 questions
adrioz
 
1100 MCQ in Dentistry with Answers
1100 MCQ in Dentistry with Answers1100 MCQ in Dentistry with Answers
1100 MCQ in Dentistry with Answers
Mahmoud Shaheen
 

Viewers also liked (6)

Mcq pharmacology
Mcq pharmacologyMcq pharmacology
Mcq pharmacology
 
Medical student surgery osce
Medical student surgery osceMedical student surgery osce
Medical student surgery osce
 
500 single best answers in medicine
500 single best answers in medicine500 single best answers in medicine
500 single best answers in medicine
 
Dentistry mcqs
Dentistry mcqsDentistry mcqs
Dentistry mcqs
 
Mcq 1060 questions
Mcq 1060 questionsMcq 1060 questions
Mcq 1060 questions
 
1100 MCQ in Dentistry with Answers
1100 MCQ in Dentistry with Answers1100 MCQ in Dentistry with Answers
1100 MCQ in Dentistry with Answers
 

Similar to 03.sabiston surgery questions 17th ed

Krok 1 - 2008 Question Paper (General medicine)
Krok 1 - 2008 Question Paper (General medicine)Krok 1 - 2008 Question Paper (General medicine)
Krok 1 - 2008 Question Paper (General medicine)
Eneutron
 
Krok 1 - 2005 Question Paper (General medicine)
Krok 1 - 2005 Question Paper (General medicine)Krok 1 - 2005 Question Paper (General medicine)
Krok 1 - 2005 Question Paper (General medicine)
Eneutron
 
Krok 1 - 2010 Question Paper (General medicine)
Krok 1 - 2010 Question Paper (General medicine)Krok 1 - 2010 Question Paper (General medicine)
Krok 1 - 2010 Question Paper (General medicine)
Eneutron
 
Krok 1 - 2015 (Biology)
Krok 1 - 2015 (Biology)Krok 1 - 2015 (Biology)
Krok 1 - 2015 (Biology)
Eneutron
 
Krok 1 - 2015 (Biochemistry)
Krok 1 - 2015 (Biochemistry)Krok 1 - 2015 (Biochemistry)
Krok 1 - 2015 (Biochemistry)
Eneutron
 
Krok 1 - 2013 Question Paper (General medicine)
Krok 1 - 2013 Question Paper (General medicine)Krok 1 - 2013 Question Paper (General medicine)
Krok 1 - 2013 Question Paper (General medicine)
Eneutron
 
Krok 1 - 2015 Base (General Medicine)
Krok 1 - 2015 Base (General Medicine)Krok 1 - 2015 Base (General Medicine)
Krok 1 - 2015 Base (General Medicine)
E_neutron
 
Krok 1 - 2015 Base (General Medicine)
Krok 1 - 2015 Base (General Medicine)Krok 1 - 2015 Base (General Medicine)
Krok 1 - 2015 Base (General Medicine)
Eneutron
 
Pnas2013
Pnas2013Pnas2013
Pnas2013
Uriel Barkai
 
Krok 1 - 2014 (Biochemistry)
Krok 1 - 2014 (Biochemistry)Krok 1 - 2014 (Biochemistry)
Krok 1 - 2014 (Biochemistry)
Eneutron
 
Krok 1 - 2007 Question Paper (General medicine)
Krok 1 - 2007 Question Paper (General medicine)Krok 1 - 2007 Question Paper (General medicine)
Krok 1 - 2007 Question Paper (General medicine)
Eneutron
 
Krok 2 - 2010 Question Paper (General Medicine)
Krok 2 - 2010 Question Paper (General Medicine)Krok 2 - 2010 Question Paper (General Medicine)
Krok 2 - 2010 Question Paper (General Medicine)
Eneutron
 
Krok 1 - 2012 Question Paper (General medicine)
Krok 1 - 2012 Question Paper (General medicine)Krok 1 - 2012 Question Paper (General medicine)
Krok 1 - 2012 Question Paper (General medicine)
Eneutron
 
Krok 1 - 2015 (Histology)
Krok 1 - 2015 (Histology)Krok 1 - 2015 (Histology)
Krok 1 - 2015 (Histology)
Eneutron
 
Krok 1 - 2014 (Path-Physiology)
Krok 1 - 2014 (Path-Physiology)Krok 1 - 2014 (Path-Physiology)
Krok 1 - 2014 (Path-Physiology)
Eneutron
 
Krok 2 - 2007 Question Paper (General Medicine)
Krok 2 - 2007 Question Paper (General Medicine)Krok 2 - 2007 Question Paper (General Medicine)
Krok 2 - 2007 Question Paper (General Medicine)
Eneutron
 

Similar to 03.sabiston surgery questions 17th ed (20)

Krok 1 - 2008 Question Paper (General medicine)
Krok 1 - 2008 Question Paper (General medicine)Krok 1 - 2008 Question Paper (General medicine)
Krok 1 - 2008 Question Paper (General medicine)
 
Krok 1 - 2005 Question Paper (General medicine)
Krok 1 - 2005 Question Paper (General medicine)Krok 1 - 2005 Question Paper (General medicine)
Krok 1 - 2005 Question Paper (General medicine)
 
2009
20092009
2009
 
Krok 1 - 2010 Question Paper (General medicine)
Krok 1 - 2010 Question Paper (General medicine)Krok 1 - 2010 Question Paper (General medicine)
Krok 1 - 2010 Question Paper (General medicine)
 
Krok 1 - 2015 (Biology)
Krok 1 - 2015 (Biology)Krok 1 - 2015 (Biology)
Krok 1 - 2015 (Biology)
 
Krok 1 - 2015 (Biochemistry)
Krok 1 - 2015 (Biochemistry)Krok 1 - 2015 (Biochemistry)
Krok 1 - 2015 (Biochemistry)
 
Krok 1 - 2013 Question Paper (General medicine)
Krok 1 - 2013 Question Paper (General medicine)Krok 1 - 2013 Question Paper (General medicine)
Krok 1 - 2013 Question Paper (General medicine)
 
Krok 1 - 2015 Base (General Medicine)
Krok 1 - 2015 Base (General Medicine)Krok 1 - 2015 Base (General Medicine)
Krok 1 - 2015 Base (General Medicine)
 
Krok 1 - 2015 Base (General Medicine)
Krok 1 - 2015 Base (General Medicine)Krok 1 - 2015 Base (General Medicine)
Krok 1 - 2015 Base (General Medicine)
 
Pnas2013
Pnas2013Pnas2013
Pnas2013
 
Krok 1 - 2014 (Biochemistry)
Krok 1 - 2014 (Biochemistry)Krok 1 - 2014 (Biochemistry)
Krok 1 - 2014 (Biochemistry)
 
2008
20082008
2008
 
Krok 1 - 2007 Question Paper (General medicine)
Krok 1 - 2007 Question Paper (General medicine)Krok 1 - 2007 Question Paper (General medicine)
Krok 1 - 2007 Question Paper (General medicine)
 
Krok 2 - 2010 Question Paper (General Medicine)
Krok 2 - 2010 Question Paper (General Medicine)Krok 2 - 2010 Question Paper (General Medicine)
Krok 2 - 2010 Question Paper (General Medicine)
 
2007
20072007
2007
 
Krok 1 - 2012 Question Paper (General medicine)
Krok 1 - 2012 Question Paper (General medicine)Krok 1 - 2012 Question Paper (General medicine)
Krok 1 - 2012 Question Paper (General medicine)
 
Krok 1 - 2015 (Histology)
Krok 1 - 2015 (Histology)Krok 1 - 2015 (Histology)
Krok 1 - 2015 (Histology)
 
Krok 1 - 2014 (Path-Physiology)
Krok 1 - 2014 (Path-Physiology)Krok 1 - 2014 (Path-Physiology)
Krok 1 - 2014 (Path-Physiology)
 
crash mcq 1.pptx
crash mcq 1.pptxcrash mcq 1.pptx
crash mcq 1.pptx
 
Krok 2 - 2007 Question Paper (General Medicine)
Krok 2 - 2007 Question Paper (General Medicine)Krok 2 - 2007 Question Paper (General Medicine)
Krok 2 - 2007 Question Paper (General Medicine)
 

Recently uploaded

New Drug Discovery and Development .....
New Drug Discovery and Development .....New Drug Discovery and Development .....
New Drug Discovery and Development .....
NEHA GUPTA
 
Novas diretrizes da OMS para os cuidados perinatais de mais qualidade
Novas diretrizes da OMS para os cuidados perinatais de mais qualidadeNovas diretrizes da OMS para os cuidados perinatais de mais qualidade
Novas diretrizes da OMS para os cuidados perinatais de mais qualidade
Prof. Marcus Renato de Carvalho
 
Ozempic: Preoperative Management of Patients on GLP-1 Receptor Agonists
Ozempic: Preoperative Management of Patients on GLP-1 Receptor Agonists  Ozempic: Preoperative Management of Patients on GLP-1 Receptor Agonists
Ozempic: Preoperative Management of Patients on GLP-1 Receptor Agonists
Saeid Safari
 
Prix Galien International 2024 Forum Program
Prix Galien International 2024 Forum ProgramPrix Galien International 2024 Forum Program
Prix Galien International 2024 Forum Program
Levi Shapiro
 
How STIs Influence the Development of Pelvic Inflammatory Disease.pptx
How STIs Influence the Development of Pelvic Inflammatory Disease.pptxHow STIs Influence the Development of Pelvic Inflammatory Disease.pptx
How STIs Influence the Development of Pelvic Inflammatory Disease.pptx
FFragrant
 
Physiology of Special Chemical Sensation of Taste
Physiology of Special Chemical Sensation of TastePhysiology of Special Chemical Sensation of Taste
Physiology of Special Chemical Sensation of Taste
MedicoseAcademics
 
HOT NEW PRODUCT! BIG SALES FAST SHIPPING NOW FROM CHINA!! EU KU DB BK substit...
HOT NEW PRODUCT! BIG SALES FAST SHIPPING NOW FROM CHINA!! EU KU DB BK substit...HOT NEW PRODUCT! BIG SALES FAST SHIPPING NOW FROM CHINA!! EU KU DB BK substit...
HOT NEW PRODUCT! BIG SALES FAST SHIPPING NOW FROM CHINA!! EU KU DB BK substit...
GL Anaacs
 
Ophthalmology Clinical Tests for OSCE exam
Ophthalmology Clinical Tests for OSCE examOphthalmology Clinical Tests for OSCE exam
Ophthalmology Clinical Tests for OSCE exam
KafrELShiekh University
 
The Normal Electrocardiogram - Part I of II
The Normal Electrocardiogram - Part I of IIThe Normal Electrocardiogram - Part I of II
The Normal Electrocardiogram - Part I of II
MedicoseAcademics
 
heat stroke and heat exhaustion in children
heat stroke and heat exhaustion in childrenheat stroke and heat exhaustion in children
heat stroke and heat exhaustion in children
SumeraAhmad5
 
Couples presenting to the infertility clinic- Do they really have infertility...
Couples presenting to the infertility clinic- Do they really have infertility...Couples presenting to the infertility clinic- Do they really have infertility...
Couples presenting to the infertility clinic- Do they really have infertility...
Sujoy Dasgupta
 
TEST BANK for Operations Management, 14th Edition by William J. Stevenson, Ve...
TEST BANK for Operations Management, 14th Edition by William J. Stevenson, Ve...TEST BANK for Operations Management, 14th Edition by William J. Stevenson, Ve...
TEST BANK for Operations Management, 14th Edition by William J. Stevenson, Ve...
kevinkariuki227
 
263778731218 Abortion Clinic /Pills In Harare ,
263778731218 Abortion Clinic /Pills In Harare ,263778731218 Abortion Clinic /Pills In Harare ,
263778731218 Abortion Clinic /Pills In Harare ,
sisternakatoto
 
How to Give Better Lectures: Some Tips for Doctors
How to Give Better Lectures: Some Tips for DoctorsHow to Give Better Lectures: Some Tips for Doctors
How to Give Better Lectures: Some Tips for Doctors
LanceCatedral
 
Pharynx and Clinical Correlations BY Dr.Rabia Inam Gandapore.pptx
Pharynx and Clinical Correlations BY Dr.Rabia Inam Gandapore.pptxPharynx and Clinical Correlations BY Dr.Rabia Inam Gandapore.pptx
Pharynx and Clinical Correlations BY Dr.Rabia Inam Gandapore.pptx
Dr. Rabia Inam Gandapore
 
New Directions in Targeted Therapeutic Approaches for Older Adults With Mantl...
New Directions in Targeted Therapeutic Approaches for Older Adults With Mantl...New Directions in Targeted Therapeutic Approaches for Older Adults With Mantl...
New Directions in Targeted Therapeutic Approaches for Older Adults With Mantl...
i3 Health
 
micro teaching on communication m.sc nursing.pdf
micro teaching on communication m.sc nursing.pdfmicro teaching on communication m.sc nursing.pdf
micro teaching on communication m.sc nursing.pdf
Anurag Sharma
 
ARTIFICIAL INTELLIGENCE IN HEALTHCARE.pdf
ARTIFICIAL INTELLIGENCE IN  HEALTHCARE.pdfARTIFICIAL INTELLIGENCE IN  HEALTHCARE.pdf
ARTIFICIAL INTELLIGENCE IN HEALTHCARE.pdf
Anujkumaranit
 
BENIGN PROSTATIC HYPERPLASIA.BPH. BPHpdf
BENIGN PROSTATIC HYPERPLASIA.BPH. BPHpdfBENIGN PROSTATIC HYPERPLASIA.BPH. BPHpdf
BENIGN PROSTATIC HYPERPLASIA.BPH. BPHpdf
DR SETH JOTHAM
 
basicmodesofventilation2022-220313203758.pdf
basicmodesofventilation2022-220313203758.pdfbasicmodesofventilation2022-220313203758.pdf
basicmodesofventilation2022-220313203758.pdf
aljamhori teaching hospital
 

Recently uploaded (20)

New Drug Discovery and Development .....
New Drug Discovery and Development .....New Drug Discovery and Development .....
New Drug Discovery and Development .....
 
Novas diretrizes da OMS para os cuidados perinatais de mais qualidade
Novas diretrizes da OMS para os cuidados perinatais de mais qualidadeNovas diretrizes da OMS para os cuidados perinatais de mais qualidade
Novas diretrizes da OMS para os cuidados perinatais de mais qualidade
 
Ozempic: Preoperative Management of Patients on GLP-1 Receptor Agonists
Ozempic: Preoperative Management of Patients on GLP-1 Receptor Agonists  Ozempic: Preoperative Management of Patients on GLP-1 Receptor Agonists
Ozempic: Preoperative Management of Patients on GLP-1 Receptor Agonists
 
Prix Galien International 2024 Forum Program
Prix Galien International 2024 Forum ProgramPrix Galien International 2024 Forum Program
Prix Galien International 2024 Forum Program
 
How STIs Influence the Development of Pelvic Inflammatory Disease.pptx
How STIs Influence the Development of Pelvic Inflammatory Disease.pptxHow STIs Influence the Development of Pelvic Inflammatory Disease.pptx
How STIs Influence the Development of Pelvic Inflammatory Disease.pptx
 
Physiology of Special Chemical Sensation of Taste
Physiology of Special Chemical Sensation of TastePhysiology of Special Chemical Sensation of Taste
Physiology of Special Chemical Sensation of Taste
 
HOT NEW PRODUCT! BIG SALES FAST SHIPPING NOW FROM CHINA!! EU KU DB BK substit...
HOT NEW PRODUCT! BIG SALES FAST SHIPPING NOW FROM CHINA!! EU KU DB BK substit...HOT NEW PRODUCT! BIG SALES FAST SHIPPING NOW FROM CHINA!! EU KU DB BK substit...
HOT NEW PRODUCT! BIG SALES FAST SHIPPING NOW FROM CHINA!! EU KU DB BK substit...
 
Ophthalmology Clinical Tests for OSCE exam
Ophthalmology Clinical Tests for OSCE examOphthalmology Clinical Tests for OSCE exam
Ophthalmology Clinical Tests for OSCE exam
 
The Normal Electrocardiogram - Part I of II
The Normal Electrocardiogram - Part I of IIThe Normal Electrocardiogram - Part I of II
The Normal Electrocardiogram - Part I of II
 
heat stroke and heat exhaustion in children
heat stroke and heat exhaustion in childrenheat stroke and heat exhaustion in children
heat stroke and heat exhaustion in children
 
Couples presenting to the infertility clinic- Do they really have infertility...
Couples presenting to the infertility clinic- Do they really have infertility...Couples presenting to the infertility clinic- Do they really have infertility...
Couples presenting to the infertility clinic- Do they really have infertility...
 
TEST BANK for Operations Management, 14th Edition by William J. Stevenson, Ve...
TEST BANK for Operations Management, 14th Edition by William J. Stevenson, Ve...TEST BANK for Operations Management, 14th Edition by William J. Stevenson, Ve...
TEST BANK for Operations Management, 14th Edition by William J. Stevenson, Ve...
 
263778731218 Abortion Clinic /Pills In Harare ,
263778731218 Abortion Clinic /Pills In Harare ,263778731218 Abortion Clinic /Pills In Harare ,
263778731218 Abortion Clinic /Pills In Harare ,
 
How to Give Better Lectures: Some Tips for Doctors
How to Give Better Lectures: Some Tips for DoctorsHow to Give Better Lectures: Some Tips for Doctors
How to Give Better Lectures: Some Tips for Doctors
 
Pharynx and Clinical Correlations BY Dr.Rabia Inam Gandapore.pptx
Pharynx and Clinical Correlations BY Dr.Rabia Inam Gandapore.pptxPharynx and Clinical Correlations BY Dr.Rabia Inam Gandapore.pptx
Pharynx and Clinical Correlations BY Dr.Rabia Inam Gandapore.pptx
 
New Directions in Targeted Therapeutic Approaches for Older Adults With Mantl...
New Directions in Targeted Therapeutic Approaches for Older Adults With Mantl...New Directions in Targeted Therapeutic Approaches for Older Adults With Mantl...
New Directions in Targeted Therapeutic Approaches for Older Adults With Mantl...
 
micro teaching on communication m.sc nursing.pdf
micro teaching on communication m.sc nursing.pdfmicro teaching on communication m.sc nursing.pdf
micro teaching on communication m.sc nursing.pdf
 
ARTIFICIAL INTELLIGENCE IN HEALTHCARE.pdf
ARTIFICIAL INTELLIGENCE IN  HEALTHCARE.pdfARTIFICIAL INTELLIGENCE IN  HEALTHCARE.pdf
ARTIFICIAL INTELLIGENCE IN HEALTHCARE.pdf
 
BENIGN PROSTATIC HYPERPLASIA.BPH. BPHpdf
BENIGN PROSTATIC HYPERPLASIA.BPH. BPHpdfBENIGN PROSTATIC HYPERPLASIA.BPH. BPHpdf
BENIGN PROSTATIC HYPERPLASIA.BPH. BPHpdf
 
basicmodesofventilation2022-220313203758.pdf
basicmodesofventilation2022-220313203758.pdfbasicmodesofventilation2022-220313203758.pdf
basicmodesofventilation2022-220313203758.pdf
 

03.sabiston surgery questions 17th ed

  • 1. Chapter 4: The Inflammatory Response Study Mode 1. How many cases of severe sepsis occur in the United States each year? A. 250,000 B. 500,000 C. 750,000 D. 1 million 2. Cytokines are: A. Lipid mediators secreted by lymphocytes B. Proteins that form the contractile elements of muscle cells C. Toxins secreted by bacteria D. Small hormone-like proteins secreted by individual cells 3. Which of the following mediators is a Th2 cytokine? A. TNF B. IL-1β C. IL-10 D. IL-12 4. Which of the following receptors is essential for activation of macrophages by bacterial lipopolysaccharide (LPS)? A. TLR4 B. IL-1RI C. IRAK D. LBP 5. Treatment with a monoclonal anti-TNF antibody has been shown to be effective for the treatment of selected cases of which of these diseases? A. Asthma B. Septic shock C. Myocardial infarction D. Crohn's disease 6. Which of the following agents has been approved by the FDA for the adjuvant treatment of severe sepsis? A. Recombinant human IL-6 B. Recombinant human activated protein C C. Recombinant human growth hormone D. Recombinant human IL-1RA
  • 2. 7. In critically ill victims of trauma, high circulating levels of IL-6 are: A. Associated with an increased risk of death B. Associated with a high likelihood of survival C. Rarely detectable D. A measurement artifact 8. Which of the following is the main cell type that is activated by IL-8? A. Enterocytes B. Macrophages C. Monocytes D. Neutrophils 9. HMGB1 is: A. A late-acting proinflammatory cytokine B. A nuclear protein that supports DNA transcription C. A protein with high electrophoretic mobility D. All of the above 10. Nitric oxide: A. Is a widely used anesthetic gas B. Is a potent endogenous vasoconstrictor C. Is generated by cells from the amino acid L-arginine D. Stimulates the aggregation of platelets
  • 3. Chapter 06: Metabolism in Surgical Patients 1. A 5-year-old boy with an unresolved congenital umbilical hernia is admitted for hernioplasty. After a moderate fasting period prior to surgery, the child is profoundly asleep and unable to be roused. Marked hypoglycemia and ketonuria, accompanied by low levels of alanine and insulin, are noted during workup. Administration of alanine produces a rapid rise in his blood glucose level. The metabolic alteration most likely causing the symptoms of this patient is expected in which of the following pathways? A. Protein breakdown in muscle tissue B. Mitochondrial β-oxidation of fatty acids by the liver C. Lipolysis by desnutrin in adipose tissue D. Glucagon secretion by alpha cells of the pancreas 2. A 52-year-old man is admitted with an episode of severe upper gastrointestinal bleeding. The patient has a 25-year history of excessive alcohol consumption and was diagnosed with cirrhosis 5 years ago. Because of recurrent and refractory bleeding from esophageal varices, he consents to an emergency portocaval shunt. After surgery, the bleeding is controlled and the patient seems to be improving until he subsequently falls into a deep coma. The best next step in the diagnostic assessment of this patient is to order which of the following tests? A. Blood glucose level B. Serum bilirubin level C. Serum albumin level D. Blood culture E. Serum ammonia level 3. Indicate which of the following is not a contraindication to enteral nutrition. A. Gastrointestinal ischemia B. Severe short bowel syndrome C. Distal high-output intestinal fistulas D. Severe acute pancreatitis 4. In the management of enteral feeding associated diarrhea, which of the following is the most appropriate initial action? A. Change of antibiotics in use. B. Change to an elemental-type formula (e.g., EleCare). C. Change to an enteral formulation with fiber. D. Start a small dose of loperamide.
  • 4. 5. In performing perioperative assessments, the most reliable biochemical predictors of operative morbidity and mortality across surgical specialties include which of the following? A. Serum albumin level B. Defects in cellular immunity and phagocytic function C. Serum urea nitrogen level D. Prothrombin time 6. A patient receives 2500 mL of Vivonex Pediatric/day, consisting of a total of 60 g of fat, 60 g of protein, and 325 g of carbohydrates. The daily caloric intake of this patient is best represented by which of the following values? A. 1600 kcal/day B. 1700 kcal/day C. 1900 kcal/day D. 2200 kcal/day 7. A previously healthy, 18-year-old man is admitted to the ICU after sustaining multiple injuries in a motorcycle accident. Multiple facial fractures, as well as ribs, pelvis, and right femur fractures, were found in the initial scan. In addition, multiple deep dermal abrasions affecting 40% of his body surface area were apparent. Nutritional support should be initiated by which of the following? A. Preferentially via the IV route B. Only via enteral feeding tube to decrease the patient's high risk of aspiration C. Immediately after resuscitation is complete (i.e., within the first 48 to 72 hours) D. After definitive treatment of the injuries has been completed 8. This question concerns the same patient as in question 7.) Two months postinjury and after operative procedures that included tracheostomy, open reductions with internal fixation, exploratory laparotomy, and persistent ventilator support, a 20% loss of body weight is noted as compared with admission. By simply considering weight loss, which of the following can be accurately expected? A. 50% increased risk of mortality B. Reduced risk for pressure sores and pneumonia C. 10% increased risk of mortality D. Increased difficulty healing and weaning of ventilator support
  • 5. 9. A 9-year-old girl suffered an 86% TBSA, third-degree burn injury during a house fire. She was found unconscious. The patient arrives to the burn unit with a heart rate (HR) of 130 beats/min, BP of 100/70 mm Hg, respiratory rate (RR) of 18 breaths/min, and temperature (T) of 37.6° C. She receives standard care with adequate IV resuscitation, thermoregulation of the room's thermostat to 33° C, and excision and grafting of her wounds on postinjury day. What is the most reliable method to estimate caloric requirements in this patient? (Questions 9 through 12 apply to this patient.) A. Harris-Benedict equation B. Curreri formula C. Indirect calorimetry D. Fick's equation 10. 7 days postinjury, HR = 165 beats/min, BP = 105/80 mm Hg, RR = 22 breaths/min, T = 38.5° C, plasma glucose level = 250 mg/dL, [Na] = 145 mEq/dL, [Cl] = 100 mEq/dL, and [K] = 4.5 mEq/dL. You administer IV fluids, insulin, and potassium chloride. What should be done to prevent mortality through the management of hyperglycemia in this critically ill surgical patient? A. Titrate patients to a glucose level goal between 80 and 110 mg/dL. B. Consider starting the patient on an oral hypoglycemic. C. Use insulin only if the plasma glucose level is >200 mg/dL D. Minimize significant fluctuations of glucose and potassium carefully. E. Request an endocrinology consultation. 11. Fifteen days following severe burns, excision, and autograft surgery, an increase of >25% of insulin requirements is noted over the previous 24-hour period. What is the best next step in the management of this patient? A. Schedule further surgery to decrease hypermetabolic response. B. Further increase the insulin drip until a glucose level of 140 to 180 mg/dL is reached. C. Order cultures and band neutrophil of peripheral blood. D. Repeat blood glucose level testing and order a new metabolic panel. 12. Four weeks postinjury, during the daily abdominal examination, you palpate the lower edge of the liver 4 cm below the edge of the ribs. No tenderness to palpation is reported and there is no evidence of jaundice. Which of the following best explains the findings seen in pathology? A. Increased dietary intake of fats B. Increased synthesis of fats from sugar-enriched diet C. Excessive peripheral lipolysis. D. Decreased β-oxidation of fat in liver mitochondria
  • 6. Chapter 07: Wound Healing 1. Fetal wound healing is different from adult wound healing in that: A. Fetal wounds heal without scarring and without dermal appendages. B. Fetal wounds have increased amounts of TGF-β and FGF-2. C. Fetal fibroblasts have decreased prolyl hydroxylase activity. D. Fetal wounds have increased responses to inflammation and growth factor stimulation. E. The ECM of the fetal wound has low levels of hyaluronic acid. 2. Elastin is: A. Organized in mammalian skin in a basket weave pattern to resist multidimensional tensile stress B. Produced late in life; has a high turnover rate C. An extremely hydrophilic molecule, which accounts for its functional properties D. An important component of the extracellular matrix of blood vessels; mutations causing elastin protein deficiency result in intimal hyperplasia, leading to arterial narrowing. E. Affected in Ehlers-Danlos syndrome, which is characterized by fragile skin 3. The cells or cell components central to wound healing are: A. B cells B. T cells C. Leukocytes D. Macrophages E. Platelets 4. Thromboxane causes: A. Fibroblast chemotaxis B. Vasoconstriction C. Collagen cross linking D. Endothelial proliferation E. Bacterial lysis 5. Iron deficiency has an impact on wound healing by decreasing: A. Early tensile strength B. DNA synthesis C. Conversion of hydroxyproline to proline D. Tissue oxygenation E. Fibroblast proliferation
  • 7. 6. Hypertrophic scar is: A. Another term for keloids B. More likely to occur on the face C. Genetic in origin D. Preventable E. Worsened with glucocorticoids 7. The effects of diabetes on wound healing include: A. Slowed epithelialization B. Reduced phagocytosis C. Glycosylated collagen D. Thickened basement membrane E. All of the above 8. Ionizing radiation causes hypoxia by: A. Direct cellular injury to endothelium B. Basal membrane injury C. Release of histamine and serotonin D. Preventing the hypoxic stimulus of angiogenesis E. Increased dermal fibrosis and thickening 9. Nicotine ingestion affects wound healing by: A. Increasing fibroblast proliferation B. Increasing platelet adhesion C. Competitively competing with oxygen D. Inhibiting oxidative metabolism E. Inhibiting oxygen transport 10. Which of the following events occurs in the proliferative phase of wound healing? A. Histamine release B. Collagen cross linking C. Thromboxane release D. Phagocytosis E. Collagen synthesis 11.Chronic wounds characteristically have: A. Tissue inflammation B. Decreased tissue inhibitor of metalloproteinases levels C. Increased gelatinase levels D. Increased collagenase levels E. All of the above
  • 8. 12. The wound healing impairment caused by corticosteroid administration can be reversed by: A. Vitamin A B. Vitamin C C. Zinc D. Vitamin K E. Vitamin B12 13. Which of the following glycosaminoglycans is not a component of skin? A. Hyaluronic acid B. Chondroitin sulfate C. Dermatan sulfate D. Heparin sulfate E. Heparin 14. Most human collagen is: A. Type I B. Type III C. Type IV D. Type V E. Type VII 15. Endothelial cells are induced to form tubules by: A. Vascular endothelial growth factor (VEGF) B. Hypoxia C. Tumor necrosis factor-α (TNF-α) D. Transforming growth factor-β (TGF-β)
  • 9. Chapter 08: Regenerative Medicine 1. Which of the following is not a primary cellular source currently being investigated for use in tissue repair? A. Embryonic stem cells B. Somatic cell nuclear transfer C. Circulating fetal stem cells D. Stromal fraction of adult bone marrow and fat E. Cancer stem cells 2. Adult mesenchymal stem cells can be characterized by: A. Their ability to undergo clonal expansion, with the ability to differentiate into fat, cartilage, and bone under appropriate conditions B. Their low frequency in fat, but significantly higher frequency and ease of harvest in bone marrow, with minimal morbidity C. The disparate growth kinetics and gene transduction capacity between fat and bone marrow sources D. The inability for bone marrow–derived cells to undergo myogenic differentiation E. A higher risk for whole blood contamination from fat-derived cells relative to bone marrow sources 3. Which of the following is not true of fetal stem cells? A. Fetal stem cells do not proliferate as fast as adult stem cells. B. Fetal stem cells have been found to possess capacity for adipogenic, osteogenic, and chondrogenic differentiation. C. Xenogeneic transplantation has shown fetal stem cells to engraft and undergo site-specific tissue differentiation. D. The use of fetal stem cells is limited by ethical debate and attendant risks associated with intrauterine procedures. 4. Which of the following is not one of the transcription factors used in cellular reprogramming to create iPS cells? A. Oct-4 B. Sox-9 C. Klf-4 D. Nanog
  • 10. 5. Which of the following is not true of ASCs? A. ASCs can be differentiated into bone, fat, and cartilage. B. The major advantage of ASCs is their relative abundance and ease of isolation from subcutaneous adipose tissue through standard lipoaspirate techniques. C. ASCs represent a homogeneous cell line derived from lipoaspirate cells. D. None of the above 6. Which of the following cell types is not pluripotent? A. ESCs B. iPS cells C. ASCs D. Cells derived by somatic cell nuclear transfer (SCNTs) 7. Induced pluripotent stem cells are characterized by: A. Requirement for viral integration of defined transcription factors to dedifferentiate into pluripotent state B. Cells that are identical to embryonic stem cells C. Cells that give rise to teratoma comprising all three germ layers when injected into immunodeficient mouse D. Inability to differentiate into neurons 8. In the skin, epidermal stem cells reside in: A. Sweat glands B. Bulge region along hair follicles C. Superficial epidermis D. Subcutaneous fat
  • 11. Chapter 09: Evidence-Based Surgery: Critically 1. The difference between efficacy and effectiveness is: A. The manner in which the outcome of interest is measured. B. Efficacy is based on best case patient care and outcomes from research settings, whereas effectiveness more closely approximates real-world results. C. Effectiveness is based on best case patient care and outcomes from research settings, whereas efficacy more closely approximates real-world results. D. No difference. 2. In performing a health economic analysis, the author(s) must be certain to describe which of the following? A. The perspective being adopted B. Discounting (to account for the future value of the dollar, usually 3% to 5%) and inflation adjustment C. The assessment of costs rather than charges D. All of the above 3. Using observational data, causality can be inferred if: A. Exposure precedes the outcome. B. The exposure can plausibly and/or biologically lead to the outcome. C. The magnitude of the association between exposure and outcome is large, with potentially varying magnitudes of association between exposure dose and outcome. D. All of the above. 4. In any study, both the hypothesis and plan of analysis should be stated a priori to decrease the risk of: A. A negative study. B. A type I error. C. A type II error. D. Low statistical power.
  • 12. 5. A group of investigators perform a randomized trial comparing a control intervention (A) to an experimental intervention (B). Their hypothesis is that intervention B is superior to A with regard to the outcome of interest. The trial is adequately powered to identify a difference if one truly exists. However, the results demonstrate no statistically significant difference between these two interventions. The investigators conclude that the two interventions are likely equivalent with regard to this outcome. What can safely be stated about the result and the authors' conclusions? A. The authors are correct; intervention B is equivalent to A in terms of the outcome of interest. B. Intervention B would likely demonstrate a statistically significant improvement over A if the sample size were larger. C. The authors are incorrect because equivalence can only be ascertained using a noninferiority trial design. D. The conclusions depend on the manner in which the authors chose to control for confounding factors. 6. Which of the following is true regarding the commonly used P value significance level of .05? A. This has been mathematically shown to be the cutoff for statistical significance. B. It is commonly used and should therefore always be chosen as the cutoff for statistical significance. C. If a statistical test attains this level of significance, it definitively proves a difference exists. D. It is arbitrary and a different level for statistical significance can be selected. 7. In analyzing data from a randomized trial, which of the following analytic approaches is most appropriate? A. Per-protocol B. Case-complete C. Intent to treat D. Meta-analysis 8. Which of the following is true when comparing the odds with the probability of an outcome? A. The odds can overestimate the probability if the outcome of interest is common in the underlying population. B. The odds can underestimate the probability if the outcome of interest is common in the underlying population. C. The odds can overestimate or underestimate the probability if the outcome of interest is common in the underlying population. D. The two values are the same.
  • 13. Chapter 13: Surgical Complications 1. Which of the following are mechanisms for heat loss that contribute to the development of hypothermia? A .Cool environment B. Direct body contact to cooler materials C. Heat loss with evaporated water vapor D. Exhalation of warmed air E. All the above 2. Which of the following is required to make a definitive diagnosis of malignant hyperthermia? A. Administration of an epidural anesthetic B. Tachycardia C. Cyanosis D. Muscle biopsy E. Muscle rigidity 3. Which of the following is not usually required for the clinical diagnosis of pneumonia? A. Decreased breath sounds B. Temperature that is usually greater than 38.5°C C. Pleuritic chest pain with coughing D. Elevated peripheral white blood cell count E. Chest radiographic infiltrate 4. Which of the following criteria is indicative of the presence of acute lung injury (ALI) rather than adult respiratory distress syndrome (ARDS)? A. Pulmonary capillary wedge pressure < 20 mm Hg B. PaO2/FIO2 ratio < 300 C. PaO2/FIO2 ratio < 200 D. Bilateral infiltrates on chest radiography E. An acute change in lung function 5. What is the expected reinfarction rate for patients undergoing noncardiac surgery after a recent acute myocardial infarction (AMI)? A. No difference compared with other myocardial infarction patients B. 8% at less than 3 months, 3.5% at 3 to 6 months, then similar to other myocardial infarction patients C. 20% in the first 6 months and then similar to other myocardial infarction patients D. 15% at less than 3 months, 10% at 3 to 6 months, 5% at more than 6 months E. 50% at less than 3 months, 25% at 3 to 6 months, 15% at more than 6 months
  • 14. 6. Components of the syndrome of inappropriate secretion of antidiuretic hormone (SIADH) include: A. Hyponatremia B. Hypernatremia C. Peripheral edema D. Serum hyperosmolality E. Hypertension 7. Which of the following is not a common cause of a small bowel obstruction in an adult? A. Hernia B. Tumor C. Volvulus D. Adhesions E. Ileocolic intussusception 8. An abdominal compartment syndrome produces all the following except: A. Acute renal failure B. Hypoxia C. Intestinal obstruction D. Elevated urinary bladder pressure E. Hypercarbia 9. Intital treatment of acute gastrointestinal bleeding includes: A. H2 receptor antagonists B. Aggressive volume resuscitation C. Gastrointestinal endoscopy D. Sucralfate E. Antibiotics 10. Surgical antibiotic prophylaxis is indicated: A. In all emergency operations B. For wounds classified as clean-contaminated C. For wounds classified as contaminated D. For wounds classified as dirty-infected E. For all wounds 11. Postrenal causes of acute renal failure include all the following except: A. Ureteral obstruction caused by stones B. Bladder dysfunction caused by nerve injury C. Urethral obstruction caused by prostatic enlargement D. A blocked Foley catheter E. Myoglobinuria
  • 15. Chapter 14: Surgery in the Geriatric Patient 1. Which of the following statements is true? A. Life expectancy of a patient 90 years old, who has no comorbid conditions, is 6 months. B. The proportion of individuals 65 years old and older will remain constant over the next 50 years. C. Age is frequently a risk factor in predicting postoperative morbidity and mortality. D. There is a decline in physiologic function in all organ systems with aging. E. It is estimated that approximately 25% of patients in most general surgical practices are older than 65 years. 2. Which of the following statements is false? A. Systolic cardiac function decreases with age at a constant rate. B. Predictable decreases in renal function occur with age and relate specifically to the glomerular filtration rate. C. Insulin secretion from the beta cell decreases as a function of age. D. Forced vital capacity (FVC) and forced expiratory volume in 1 second (FEV1) decrease steadily with aging. E. The synthetic capacity of the liver, as measured by standard liver function tests, remains unchanged with age. 3. Which of the following is not a necessary part of the preoperative assessment in an older adult? A. Baseline cognitive evaluation B. Exercise capacity testing C. Determination of nutritional status D. Advance directives and discussion of end-of-life wishes E. Evaluation of ADLs and IADLs 4. In emergency surgery for perforated ulcer disease, a patient with preoperative shock, more than 48 hours of perforation, and significant comorbid disease has a mortality rate close to: A. 0% B. 25% C. 50% D. 75% E. 100%
  • 16. 5. Which of the following is true regarding appendicitis in older adults ? A. Most patients present with classic signs and symptoms of right lower quadrant pain, increased white blood cell count, and fever. B. Although there are typically delays in presentation to the hospital, the diagnosis is usually made in a timely fashion. C. Of these patients, 18% present with no abdominal pain. D. Reported rates of perforated appendicitis in octogenarians are less than 50%. E. The overall mortality in patients older than age 65 with appendicitis is approximately 18%. 6. In an older patient with suspected biliary disease, which of the following statements is false? A. The rate of gallstones is 30% to 40% of individuals older than 80 years. B. There is an increased incidence of common bile duct stones in patients undergoing cholecystectomy. C. Gallbladder motility is typically normal. D. The conversion rate from laparoscopic cholecystectomy to open procedures is higher than in younger patients. E. Biliary tract disease is a frequent cause of acute abdominal complaints in older adults. 7. Which of the following statements is not true? A. Breast cancer in older women is more frequently associated with the presence of favorable tumor markers. B. Stage per stage, survival for older women with breast cancer is better than that seen in younger women. C. Breast cancer trials in the United States have a disproportionately low enrollment of older women. D. Advanced age is not a contraindication to breast-conserving surgery. E. older women have an increased incidence of significant side effects from axillary lymph node dissection (ALND). 8. Which of the following is not currently recommended for an older surgical patient? A. Carotid endarterectomy B. Carotid angioplasty and stenting C. Open abdominal aortic aneurysm repair D. Endovascular aneurysm repair E. Peripheral arterial bypass
  • 17. Chapter 15: Morbid Obesity 1. Hormones or peptides involved in satiety include: A. Gastrin B. Somatostatin C. Glucagon D. Ghrelin E. Estrogen 2. Currently accepted guidelines from the National Institutes of Health for preoperative selection of patients for weight reduction surgery include all the following except: A. Patients with Prader-Willi syndrome B. BMI > 35 kg/m2 with associated medical comorbidity worsened by obesity C. Failed medical therapy D. Psychiatrically stable E. Motivated patient 3. Absolute contraindications for bariatric surgery include: A. Cardiomyopathy B. Pickwickian syndrome C. Type 1 diabetes mellitus D. Nonalcoholic steatotic hepatitis E. None of the above 4. Laparoscopic sleeve gastrectomy has: A. A higher leak rate than laparoscopic RYGB B. A lower mortality rate for super obese BMI > 60 kg/m2 undergoing duodenal switch C. A theoretical advantage over RYGB for iron absorption D. All of the above E. None of the above 5. One specific problem that may arise with persistent vomiting after any of the bariatric operations is Wernicke's encephalopathy, which can be treated with parenteral: A. Vitamin B12 B. Omeprazole C. Thiamine (vitamin B1) D. Ascorbic acid (vitamin C) E. Scopolamine
  • 18. 6. Morbid obesity is defined as: A. 1.5 times ideal body weight B. BMI > 40 kg/mg2 C. 20% above ideal body weight for adolescents D. A function of physical activity, comorbid conditions, and weight E. Weight > 40 kg 7. Bariatric operative procedures that produce weight loss by a combination of restriction of oral intake and malabsorption include all the following except: A. Vertical banded gastroplasty B. Duodenal switch C. Biliopancreatic diversion D. Roux-en-Y gastric bypass E. None of the above 8. Long-term metabolic complications of Roux-en-Y gastric bypass include: A. Hyperlipidemia B. Vitamin C deficiency C. Vitamin K deficiency D. Lactic acidosis E. Iron deficiency 9. Medical therapy that has proven to have significant long-term success in morbidly obese patients includes: A. Low-calorie diets B. Sibutramine C. Ghrelin D. Orlistat E. None of the above 10. Produced in the proximal stomach, levels of the hormone ghrelin: A. When decreased, seem to produce increased food intake B. Are suppressed in postoperative patients who have undergone gastric bypass C. Are decreased in individuals on a low-calorie diet D. Regulate release of insulin E. Are decreased in patients after adjustable gastric banding.
  • 19. Chapter 16: Anesthesiology Principles, Pain 1. Which of the following statements is true regarding isoflurane compared with halothane? A. Isoflurane is associated with smoother inhalational induction. B. Isoflurane produces greater sensitization to the arrhythmogenic effects of catecholamines. C. Isoflurane has greater potency, as reflected in a lower minimal alveolar concentration (MAC). D. Isoflurane is associated with more rapid emergence. E. Isoflurane increases bronchoconstriction, whereas halothane decreases bronchoconstriction. 2. Which of the following drugs is useful as a premedicant because of its potent amnesic effects? A. Glycopyrrolate B. Etomidate C. Midazolam D. Ketamine E. Thiopental 3. Which of the following statements most accurately describes the differences between subarachnoid block and epidural block? A. Subarachnoid block is associated with more rapid onset of hypotension. B. Subarachnoid block is associated with greater risk of systemic local anesthetic toxicity. C. Subarachnoid block is associated with less risk of post–lumbar puncture headache. D. Subarachnoid block should not be performed with a mixture of local anesthetic and opioids. E. Subarachnoid block is associated with a small risk of cardiac arrest for which resuscitation is uncomplicated. 4. Essential monitors for all anesthetics include which of the following? A. Direct arterial pressure B. Exhaled volatile anesthetic concentration C. Bioimpedance cardiac output D. Electrocardiography E. Anesthetic depth (bispectral index)
  • 20. 5. Which of the following muscle relaxants is largely metabolized by Hofman degradation in plasma and is relatively independent of renal elimination? A. Pancuronium B. Vecuronium C. Cisatracurium D. Rocuronium E. Atracurium 6. Which of the following statements is accurate regarding preoperative cardiac evaluation? A. Older patients undergoing cataract surgery require preoperative stress testing. B. Patients who require major vascular surgery should undergo cardiac catheterization before scheduling the vascular procedure. C. Patients can be stratified for the need for cardiac evaluation based on symptoms and magnitude of the anticipated surgery. D. Patients with previous myocardial revascularization are at high risk for perioperative myocardial infarction. E. Ambulatory electrocardiography is sensitive and specific for the identification of patients at high perioperative risk. 7. Which of the following guidelines is recommended for preoperative fasting? A. Aspiration of gastric contents is not a problem; all patients can take food and liquids freely until immediately before surgery. B. A fasting period of 2 hours or more is recommended after ingestion of clear liquids. C. All patients should not receive food or liquids after midnight before surgery the next day. D. A fasting period of 6 hours or more is recommended after ingestion of solids. E. Both B and D 8. Which of the following statements is accurate regarding local anesthetic toxicity? A. The earliest symptoms are referable to the central nervous system. B. At the first sign of local anesthetic toxicity, succinylcholine should be given. C. When local anesthetics are used for regional block, adding epinephrine decreases the toxic dose. D. Bupivacaine is the least toxic of currently used local anesthetics. E. Ester-type local anesthetic agents are more toxic than amide agents because of slow metabolism.
  • 21. 9. Which of the following problems is commonly recognized in the postanesthesia care unit (PACU)? A. Delirium B. Emesis C. Hypoxemia D. Hypertension E. All of the above 10. Characteristics of moderate sedation include which of the following? A. Absence of movement in response to a skin incision B. Preserved airway reflexes C. Motor response only to painful stimuli D. Moderate respiratory depression E. Moderate hypotension 11. Which four steps accurately describe the process of nociception? A. Transduction, transmission, modulation, and perception B. Recognition, registration, amplification, and interpretation C. Perception, integration, orientation, and implementation D. Description, analysis, formulation, and recognition E. Deformation, translation, registration, and formulation 12. Which of the following opioids is partially converted to a metabolite that can accumulate and cause seizures in patients with renal impairment? A. Fentanyl B. Hydromorphone C. Codeine D. Morphine E. Meperidine 13 .What is the correct term for the physiologic process in which a previously effective dose of an opioid fails to provide adequate analgesia? A. Addiction B. Psychological dependence C. Physical dependence D. Tolerance E. Malingering
  • 22. 14. Compared with patients who receive conventional, intermittent, nurse-administered opioid delivery, patients who receive intravenous patient-controlled analgesia experience which of the following advantages? A. Prompt analgesia B. Smaller doses of opioids C. Better maintenance of blood concentration of drugs in the analgesic range D. Lower incidence of drug-related side effects E. All of the above
  • 23. Chapter 19: The Difficult Abdominal Wall 1. Abbreviated laparotomy is the initial phase of damage control surgery. The indications are as follows: A. Temperature less than 35° C B. Medical bleeding C. Arterial pH less than 7.20 D. Urine output less than 30 mL/kg/hr E. A, B, and C are correct 2. The most common indications for the use of the open abdomen technique in general surgery are as follows: A. Abdominal compartment syndrome B. Ruptured abdominal aortic aneurysm C. Trauma-damage control D. Acute pancreatitis E. All of the above 3. If not recognized and treated, intra-abdominal hypertension can progress to abdominal compartment syndrome. Which of the following clinical signs are hallmarks of abdominal compartment syndrome? A. Oliguria B. Metabolic alkalosis C. Increased peak inspiratory pressures D. Intestinal ileus E. A and C are correct 4. There are several techniques for creating a temporary abdominal closure for the open abdomen. The key to all techniques must include the following: A. Quick application B. Seal in moisture and temperature C. Quickly removable D. High tensile strength E. A, B, and C are correct
  • 24. 5. During the staged abdominal repair phase of damage control surgery, the surgeon has several challenging questions to answer on return to the operating room. Which of the following approaches can be used to address small and large bowel injuries? A. Resection of devitalized tissue B. Primary repair C. Externalization with creation of a stoma D. Primary bowel anastomosis E. All of the above 6. The open abdomen technique has a high rate of nonclosure because of the following complications: A. Intra-abdominal abscess and intra-abdominal sepsis B. Acute lung injury C. Atmospheric intestinal fistula D. Urinary tract infection E. A and C are correct 7. Which of the following mesh products should not be used in the open abdomen setting because they have very high rates of intestinal fistula formation and mesh infection? A. Human dermal acellular dermis B. PTFE C. Porcine dermal matrix D. Polypropylene E. B and D are correct 8. The decision to close an open abdomen with visceral edema can be complicated. Which of the following physiologic criteria can be used to guide abdominal closure in the operating room? A. Decrease in urine output by 10 mL/hr B. Change in peak inspiratory airway pressure less than 10 cm H2O while attempting to bring the fascia together C. Increase in intracranial pressure by 5 cm H2O D. Sustained increase in intra-abdominal pressure less than 20 mm Hg E. B and C are correct
  • 25. Chapter 20: Emergency Care of Musculoskeletal 1. Which of the following will increase the stiffness of an external fixation construct? A. Using stainless steel instead of titanium pins B. Using more pins C. Placing the bars closer to the bone D. Placing the bars in multiple planes E. All of the above 2. A patient is found to have an isolated fracture of the medial malleolus on an ankle series x-ray. What other imaging should be performed? A. AP, lateral, and oblique views of the foot to look for a fifth metatarsal fracture. B. CT scanning of the ankle to look for a tibial plafond fracture C. AP and lateral views of the tibia and fibula to look for a proximal fibular fracture D. AP and cross-table lateral views of the hip to look for a femoral neck fracture E. PA and lateral views of the lumbar spine to look for a lumbar burst fracture 3. An absolute indication to perform a four-compartment fasciotomy of the leg for compartment syndrome is: A. Firm compartments on physical examination B. ∆P < 30 mm Hg C. Subjective complaints of paresthesias in the foot D. Severe leg pain E. Unconscious patient with a tibial shaft fracture 4. A 36-year-old man presents to the trauma bay after a motorcycle crash. His systolic pressures remain in the 70s despite resuscitation with packed red blood cells and crystalloid. His chest x-ray is normal. His pelvic x-ray is shown in Figure 20-30. What is the next most appropriate treatment? A. Application of a pelvic binder B. Placement of a chest tube C. CT of chest, abdomen, and pelvis D. Emergent exploratory laparotomy E. Arteriography
  • 26. 5. A 26-year-old man presents to the emergency department with the fracture seen in Figure 20-4A. A Hare traction splint was placed in the field. Traction on the injured limb should be maintained. Which of the following is the most appropriate method of traction for this patient? A.Skin traction with a Buck boot B. Distal femoral traction pin placed from medial to lateral C.Proximal tibial traction pin placed from medial to lateral D.Proximal tibial traction pin placed from lateral to medial E. Hare traction splint left in place 6. A 47-year-old woman sustains the fracture shown in Figure 20-40. Which examination tests the nerve most commonly injured with this fracture pattern? A. Shoulder abduction B. Elbow flexion C. Wrist extension D. Wrist flexion E. Finger flexion
  • 27. Chapter 21: Burns 1. The zone of stasis in a burn wound is associated with which of the following? A. Direct thermal damage B. Vasodilation C. Neutrophil adherence D. Platelet degranulation E. Non-nutrient shunting 2. Deep second-degree wounds reepithelialize from retained keratinocytes in: A.Rete ridges B. Hair follicles C. Moll glands D. Reticular dermis E. Meissner corpuscles 3. A patient with burns to the entire back, scalp (50% of the head and neck), and posterior thighs has what percentage of his or her total body surface area (TBSA) burned? A.40% B.28% C.20% D.32% E.36% 4. Severe burns are associated with which of the following immunodeficiencies in the acute phase? A. Neutropenia B. Granulocyte colony-stimulating factor deficiency C. Decreased cytotoxic T cell activity D. Increased neutrophil apoptosis E. Antibody overproduction 5. After major burn injury, the metabolic changes are characterized by an ebb and flow phase. Changes consistent with the ebb phase include: A. Increased oxygen delivery B. Low cardiac output C. Hypermetabolism D. Hyperthermia E. Hyperglycemia
  • 28. 6. Which of these therapeutic approaches to attenuate the hypermetabolic response can lead to hyperglycemia? A. Insulin B. Growth hormone C. Metformin D. PPAR-γ agonists E. Oxandralone 7. A 40-year-old, 100-kg man is involved in a house fire with burns to 45% of his TBSA. He comes to the emergency department with two peripheral IV lines that are not being used. It is 2 hours since his injury, and he has not received any resuscitation. His initial IV fluid rate should be: A. 250 mL/hr B. 500 mL/hr C. 1000 mL/hr D. 1500 mL/hr E. 2000 mL/hr 8. The relative surface area of regions of the body used in calculating burn surface area differs in children from adults. Which of the following statements describes these differences? A. Children have relatively less surface area in the arms and legs and more in the trunk. B. Adults have relatively more surface area in the trunk and less in the upper extremities. C. Children have relatively more surface area in the head and neck and less in the lower extremities. D. Adults have more surface area in the head and neck and less in the lower extremities. E. Children have relatively less area in the head and neck and more in the trunk 9. Which of the following synthetic and biologic dressings are currently used for burn wound closure? A. Allograft B. Xenograft C. Stem cells D. Biobrane E. Integra 10. Injury from smoke inhalation during a house fire occurs from which of the following? A. Thermal injury B. Excessive coughing C. Splinting leading to atelectasis D. Plugging of airways from concentration of soot E. Toxic chemicals in smoke particles
  • 29. 11. What are the three zones of injury after burn? A. Coagulation, stasis, necrosis B. Fibrinolysis, stasis, injury C. Coagulation, injury (stasis), hyperemia D. Edema, injury, necrosis 12. What proinflammatory mediators are responsible for the postburn hypermetabolic response? A.TNF-α, IL-1, IL-6, catecholamines, glucagon, cortisol, endotoxin, nitric oxide B. Cortisol, IL-5, IL-2, epinephrine, insulin C. IL-10, CD4, insulin, TNF-α, glucosamine D. Glucose, cortisol, norepinephrine, nitric oxide 13. One cause of multisystem organ failure after severe burn injury is: A. Decreased intestinal permeability to macromolecules B. Diminished blood volume and cardiac output C. Decreased peripheral vascular resistance D. Decreased presence of endotoxin 14. Severely burned patients with no other complications can lose 25% of total body mass after acute burn injury. This loss is associated with: A. Pneumonia and pressure ulcers B. Pneumonia and decreased wound healing C. Immune dysfunction and death D. Decreased wound healing and immune dysfunction 15. Improvements in morbidity from severe burn injury stem from: A. Decreased ambient temperature, late excision and grafting after resuscitation, and early aggressive parenteral nutrition B. Increased ambient temperature, early excision and grafting, and early enteral nutrition C. Controlling sepsis and late excision and grafting D. Selective beta blockers and high-fat and high-protein diets
  • 30. Chapter 22: Bites and Stings 1. Which of the following statements regarding snakebite management in North America is true? A. Antivenom should be administered to any patient who presents to the hospital with a bite from a definitively identified rattlesnake. B. Skin testing for possible allergy should be performed before the administration of CroFab. C. CroFab is effective for reversing venom poisoning by all North American pit vipers and coral snakes. D. Fasciotomy should be performed only in the setting of objectively measured elevation of pressures in involved muscle compartments. 2. Which of the following injuries is considered low risk for becoming infected and can be repaired by primary closure? A. Dog bite to the face B. Cat bite to the upper arm C. Human bite over the dorsum of the metacarpophalangeal joint D. Primate bite to the foot 3. Which of the following statements is false regarding cat bites? A. Pasteurella multocida is the primary organism involved in infections. B. Puncture wounds are common. C. Wound infection occurs in 10% to 20% of cat bites. D. Most cat bites are considered high risk for infection and inappropriate for primary closure. 4. Which of the following statements is false regarding rabies? A. Most patients acquiring rabies from a bat do not recall being in contact with the bat. B. Rabies is caused by rhabdovirus found in the saliva of mammals. C. Patients with preexposure rabies immunization need active immunization only. D. If diagnosed early, rabies is usually treatable. 5. Prophylactic antibiotics should be administered for all of the following injuries except: A. Cat bite to the foot B. Human bite to the ear C. Dog bite to the arm of a diabetic patient D. Dog bite to the scalp of a child
  • 31. 6. Management of spider bites should include: A. Prompt administration of antivenom for any victim of a black widow spider bite B. Application of local cooling measures and conservative wound care C. Empirical administration of dapsone for wounds believed to be consistent with brown recluse spider bites D. Early (≤1 hour) excision of the bite site to limit venom spread 7. Which of the following statements regarding Lyme disease is true? A. Patients with early Lyme disease often present with a rash consistent with erythema multiforme. B. Neurologic involvement, neuroborreliosis, occurs in approximately 80% of untreated patients. C. First-line treatment for early Lyme disease without neurologic involvement includes doxycycline for 14 to 21 days. D. Attempts to develop a safe effective vaccine for Lyme disease have so far proven unsuccessful. 8. Treatment of Hymenoptera-induced anaphylactic shock should include: A. Epinephrine B. Antihistamines (H1 and H2 blockers) C. Steroids D. Referral to an allergist for possible desensitization therapy E. All of the above 9. Treatment for a moray eel bite includes: A. Antirabies immunization B. Débridement and primary closure C. Débridement and delayed primary closure D. Administration of antivenin 10. Treatment for a sea urchin puncture includes: A. Percussion and fragmentation of the spine B. Rapid extraction of the spine, followed by wide excision C. Exploration of any discolored skin marking D. Ice water immersion E. None of the above
  • 32. Chapter 23: Surgical Critical Care 1. A patient is defined as having brain death if the patient has complete absence of cortical brain function and which of the following? A. Loss of pupillary reflex to light B. Loss of the vestibulo-ocular reflex C. Loss of oropharyngeal reflex (gag reflex) D. Apnea on consecutive tests despite adequate stimulation (PaCO2 >60 mm Hg) E. There is no one accepted definition of brain death nationally 2. The Confusion Assessment Method (CAM-ICU) is useful in determining the presence or degree of which of the following in patients in the intensive care unit? A. Chronic dementia B. Adequacy of sedation regimens in mechanically ventilated patients C. Adequacy of analgesia in postoperative surgical patients D. Identifying, evaluating, and managing acute delirium E. Managing alcohol withdrawal 3. In a mechanically ventilated patient with a traumatic brain injury or in a patient in a coma (GCS <8) in whom there is suspected elevation of intracranial pressure (ICP), what is an appropriate PaCO2 goal to maintain? A. 25 to 30 mm Hg B. 30 to 35 mm Hg C. 35 to 40 mm Hg D. 40 to 45 mm Hg E. 45 to 50 mm Hg 4. Which of the following statements is true regarding measurement of central venous pressure (CVP) in postoperative surgical patients as a guide for management of fluid resuscitation? A. Right-sided heart function is a reliable predictor of left-sided heart function. B. It is inaccurate in assessing volume status in a diverse group of surgical patients. C. Values can be interpreted regardless of the patient's degree of mechanical ventilation requirements of positive end-expiratory pressure (PEEP). D. It is a good guide for instituting and managing vasopressor agents in hypotensive patients.
  • 33. 5. A postoperative patient with new-onset unstable atrial fibrillation with systolic blood pressure of 70 mm Hg should be treated with which of the following modalities? A. Direct current cardioversion B. Intravenous beta blocker C. Intravenous calcium channel blocker D. Digoxin 6. Which of the following modalities has been shown to reduce mortality in adult patients with the acute respiratory distress syndrome (ARDS) in prospective randomized trials? A. Corticosteroid early in the course of ARDS (<7 days) B. Surfactant replacement therapy C. Nitric oxide D. Maintenance of lower filling pressure with pulmonary capillary wedge pressure (PCWP) less than 8 mm Hg E. Lung protective ventilation with low tidal volumes of 6 mL/kg of ideal body weight 7. Abdominal compartment syndrome (ACS) is best described by which of the following? A. Intra-abdominal pressure greater than 15 mm Hg B. Intra-abdominal pressure greater than 25 mm Hg and evidence of decreased end organ perfusion (i.e., oliguria, renal dysfunction, hypotension) C. Intra-abdominal pressure greater than 35 mm Hg with hypoxemia D. Peak airway pressure greater than 40 cm H2O 8. Which of the following have been shown to be clinical advantages of enteral feeding versus total parenteral nutrition (TPN) in critically ill surgical patients? A. Preservation of gut mucosal integrity and barrier function B. Secretory IgA production of the gut C. Decreased rates of catheter-related bloodstream infections D. Lower cost E. All of the above 9. Which of the following are strict indications to guide the institution of renal replacement therapy in the form of intermittent hemodialysis or continuous venovenous filtration or hemodialysis in critically ill surgical patients? A. Increasing oxygen requirement and chest x-ray findings of interstitial edema and engorged pulmonary vasculature B. Blood pH less than 7.25 C. Potassium level greater than 6.0 mEq/L D. There are no specific or strict indications to start renal replacement therapy E. CO2 level less than 16 mEq/L on chemistry profile
  • 34. 10. Which of the following is most appropriate regarding glycemic control in critically ill surgical patients? A. Goal glucose should be 80 to 110 mg/dL to improve outcomes in patients with traumatic brain injury. B. Stress-related hyperglycemia should be managed with longer acting forms of insulin such as insulin glargine. C. Maintaining glucose levels less than 180 mg/dL compared with maintaining a range of 81 to 110 mg/dL results in fewer episodes of hypoglycemia and lower mortality.
  • 35. Chapter 27: Liver Transplantation 1.The most common indication for liver transplantation in the United States is: A. EtOH B. Sclerosing cholangitis C. Hepatitis B virus D. Hepatitis C virus E. Hepatocellular carcinoma 2.An infant with extrahepatic biliary atresia, chronic liver insufficiency, and failure to thrive may be served by: A. Whole pediatric liver transplantation B. Split orthotopic liver transplantation C. Live donor liver transplantation D. All of the above E. None of the above 3.The current risk of death to the donor for live donor liver transplantation is: A. About the same as the risk to a potential kidney donor B. 1/100 C. 1/1,000 D. 1/10,000 E. 1/100,000 4.The current system of liver distribution is primarily based on: A. Insurance B. Medical necessity C. Region D. Recipient age E. None of the above 5.The liver can be divided into segments based on A. Portal vein inflow B. Hepatic artery inflow C. Biliary outflow D. Hepatic vein outflow E. All of the above 6.Which are appropriate treatment(s) for hepatocellular carcinoma and cirrhosis? A.Whole liver transplantation B. Liver resection C. Live donor liver transplantation D. Resection with salvage transplantation E. All of the above
  • 36. 7.Which statement about immunosuppression medication is false? A. Cellcept (mycophenolate) is an antiproliferation agent. B. Calcineurin inhibitors have nephrotoxic and neurotoxic side effects. C.Calcineurin inhibitors prevent expansion of the host response by inhibiting IL-10 production. D. Immunosuppressive agents are optionally used in combination to maximize their effect and minimize toxicity.
  • 37. Chapter 28: Kidney and Pancreas Transplantation 1.A 50-year-old man arrives in the recovery room following an uneventful living donor kidney transplantation. It was documented that there was good urine output in the operating room. When the patient arrives, there is no urine output. The next best step would be to: A. Order an ultrasound. B. Perform an emergent biopsy to rule out hyperacute rejection. C. Examine the patient. D. Flush the Foley catheter. 2.A 50-year-old man arrives in the recovery room following an uneventful living donor kidney transplantation. It was documented that there was good urine output in the operating room. When the patient arrives, there is no urine output. The physical exam reveals BP, 134/70 mm Hg, HR, 76 beats/min, RR, 14 breaths/min, O2 saturation, 100%, Foley is intact. An inspection of the abdomen reveals a distended right lower quadrant. The incision is dry. The next best step is to: A. Order an ultrasound. B. Open the wound at the bedside. C. Flush the Foley catheter. D. Return to the operating room. 3.A 50-year-old man arrives in the recovery room following an uneventful living donor kidney transplantation. It was documented that there was good urine output in the operating room. When the patient arrives, there is no urine output. The most likely diagnosis in this case is: A. Lymphocele B. Urine leak C. Hyperacute rejection D. Wound dehiscence E. Arterial thrombosis 4.A 35-year-old woman with type 1 diabetes wishes to be evaluated for a kidney pancreas transplantation. She has a history of hypertension and has recently started dialysis. There is no family history of cardiac disease. Her initial workup should include: A. C-peptide level B. Cardiac stress test C. Pap smear D. All of the above
  • 38. 5.A 35-year-old woman with type 1 diabetes wishes to be evaluated for a kidney pancreas transplantation. She has a history of hypertension and has recently started dialysis. There is no family history of cardiac disease. The patient presents at 3 weeks post-transplantation with an elevated amylase level and her creatinine level is also somewhat elevated at 1.4 mg/dL, from a baseline of 1.0 mg/dL. Her glucose level is normal and her urine output has been normal. Diagnostic tests should include: A. Abdominal and pelvic CT scan B. Urinalysis C. CMV polymerase chain reaction assay D. Ultrasound and kidney biopsy E. C-peptide level
  • 39. Chapter 29: Small Bowel Transplantation 1. Which of the following patients would be considered an appropriate candidate for intestinal transplantation? A. A 2-year-old boy with long-segment Hirschsprung disease extending into the jejunum with an end jejunostomy. He receives a combination of continuous tube feeds via a gastrostomy tube and supplemental intravenous fluids via a tunneled central venous catheter. He has had the central venous line replaced once because of a break in the line but has not had a known central venous line infection. B. A 23-year-old woman with a history of Crohn disease who has had multiple bowel resections because of strictures. She is currently dependent on TPN, although she can tolerate small amounts of oral intake. Her liver function is normal. She has had one central venous line infection owing to Staphylococcus epidermidis, which was successfully treated with antibiotics. C. A 1-year-old girl who was born prematurely and lost most of her small intestine secondary to necrotizing enterocolitis. She is currently dependent on TPN and has been hospitalized five times for central venous line infections, two of which were due to Candida albicans. D. A 10-year-old boy with pseudo-obstruction who has recurrent episodes of abdominal pain and distention. He has recently had to begin TPN because of failure to maintain adequate growth. 2. A 2-year-old boy with intestinal failure secondary to gastroschisis and parenteral nutrition–associated liver disease (PNALD) is on the waiting list for a combined liver-intestine transplant. His blood type is A, and he weighs 13 kg. Which of the following potential donors would be most appropriate? A. A 1-year-old donor who sustained head traumas as a result of child abuse; the donor is blood type A and weighs 8 kg B. An 8-year-old donor who sustained head trauma as a result of a bicycle versus motor vehicle accident; the donor is blood type A and weighs 25 kg C. A 20-year-old donor who sustained head trauma as a result of a motorcycle accident; the donor is blood type A and weighs 60 kg D. A 2{1/2}-year-old donor who sustained head trauma as a result of a motor vehicle accident; the donor is blood type A and weighs 14 kg 3.For patients who require a liver transplant in addition to an intestinal transplant owing to intestinal failure and parenteral nutrition–associated liver disease (PNALD), what is the advantage of including the pancreas en bloc with the other organs? A. Most patients with intestinal failure and PNALD also have diabetes. B. Including the pancreas avoids having to perform any hilar dissection in the donor organs and avoids having to perform separate vascular anastomoses for the liver and the intestine during implantation in the recipient. C. The native pancreas is removed from the recipient with the recipient's liver and remnant small intestine and needs to be replaced. D. Including the pancreas has been shown to decrease the incidence of rejection.
  • 40. 4. Which of the following is the most effective induction agent for intestinal transplantation? A. Alemtuzumab (Campath) B. Basiliximab (Simulect) C. Daclizumab (Zenapax) D. Rabbit antithymocyte globulin (Thymoglobulin) E. No agent has been proven superior to the others 5.The most effective method of monitoring an intestinal allograft for rejection is: A. Serum liver function tests B. Endoscopically obtained mucosal biopsy specimens reviewed by a pathologist C. Measuring stoma or stool output D. Clinical signs such as abdominal pain and distention 6.Infection with which of the following pathogens may mimic rejection in intestinal transplant recipients? A. Epstein-Barr virus B. Escherichia coli C. Cytomegalovirus D. Enterobacter species E. Klebsiella species 7.Recipients of which transplanted organ are at the highest risk of developing post-transplant lymphoproliferative disorder (PTLD)? A. Kidney B. Intestine C. Heart D. Liver 8.The most common cause of death after intestinal transplantation is: A. Infection B. Post-transplant lymphoproliferative disorder (PTLD) C. Graft-versus-host disease (GVHD) D. Chronic rejection
  • 41. Chapter 30: Tumor Biology and Tumor Markers 1.Which of the following genetic changes may be involved in tumorigenesis? A. Activation of a proto-oncogene B. Loss of a tumor suppressor gene C. Activation of a growth factor receptor–encoding gene D. All of the above 2.Which of the following statements is incorrect? A. Cancer is the second most common cause of death in the United States. B. Cancer disproportionally affects people 65 years and older. C. It is estimated that 15% to 50% of all cancer deaths in the United States can be attributed to overweight and obesity. D. Cancer incidence is the number of cancer patients in the population. 3.Which type of adjuvant therapy may provide the best strategy for the postoperative eradication of residual microscopic disease? A. Chemotherapy B. Radiation therapy C. Immunotherapy D. Combination therapy 4.Germline mutations have been postulated to be associated with several tumors. Which of the following is an example? A. p53 B. APC C. KRAS D. All of the above 5.Tumor growth is dependent on: A. Paracrine factors B. Autocrine and paracrine factors C. Immune cell infiltrate D. Autocrine factors 6.Which early events in the primary tumor are characteristic for the formation of metastases? A. Angiogenesis B. Intravasation and extravasation C. Evasion of cell death D. Detachment from extracellular matrix and intravasation.
  • 42. 7. What is most essential for the development of a tumor? A. Successive genetic alterations B. The ability to produce growth factors C. Deletion of p53 D. Immunosuppressed or immunodeficient host 8.An ideal tumor marker is A. Detectable early with a high degree of false-negative findings B. Detectable only when tumors metastasize C. Characterized by a high specificity and low sensitivity D. exclusively by the particular tumor 9.CA 19-9 levels may be elevated in which of the following conditions? A. Pancreatic adenocarcinoma B. Benign biliary stricture C. Malignant biliary stricture D. Colon cancer E. All of the above 10.A false-positive CEA test can occur in which of the following benign conditions? A. Ulcerative colitis B. Cirrhosis C. COPD D. Gallstone pancreatitis E. All of the above 11.Which of the following is not correct regarding the use of alpha-fetoprotein (AFP) as a tumor marker for hepatocellular carcinoma (HCC)? A. AFP levels may be elevated in other gastrointestinal malignancies. B. The combination of ultrasound with AFP improves the sensitivity of screening for HCC. C. Following complete resection, AFP levels should fall below 10 ng/mL. D. The rate at which the AFP level rises is not associated with a worse prognosis. E. None of the above. 12.Which of the following is correct regarding prostate-specific antigen (PSA)? A. PSA levels do not rise after digital rectal examination. B. PSA level > 4 ng/mL is diagnostic of prostate cancer. C. After complete resection, the PSA level should normalize after 2 to 3 weeks. D. An elevated but stable PSA level after radiotherapy does not portend clinical relapse. 13..A highly specific test for a tumor marker may still yield a large number of false-positive test results: A. When the test has low sensitivity B. When the prevalence of the disease tested for is low in the tested population C. When the test is done with urine samples instead of serum samples D. When the assay is a radioimmunoassay E. When the test has to be done on fresh tumor samples instead of preserved samples
  • 43. 14..Breast cancer specimens are now routinely tested for which of the following tumor markers? A. Estrogen receptor B. Progesterone receptor C. Her2/neu expression D. A and B E. A, B, and C 15.Her2/neu expression status of a breast tumor is important for: A. Monitoring the efficacy of therapy B. Determining treatment for recurrent cancer C. Diagnosis D. Timing of second-look procedures E. All of the above 16.Patients with metastatic colorectal cancer who lack a KRAS mutation in codon 12 or 1: A. Are more likely to respond to anti-EGFR antibody therapy B. Are less likely to respond to anti-EGFR antibody therapy C. Have improved disease-free survival when treated with anti-EGFR antibody therapy D. Have no change in overall survival when treated with anti-EGFR antibody therapy E. A and C F. A and D 17.Which of the following statements is incorrect regarding the use of the 21-gene assay, Oncotype DX, in breast cancer? A. Predicts likelihood of local tumor recurrence B. Was designed for patients with node-negative, tamoxifen-treated breast cancer C. Cannot be used in ER-negative tumors D. Alters treatment choice in approximately 25% of cases E. Assays 16 tumor-associated gene
  • 44. Chapter 32: Melanoma and Cutaneous Malignancies 1.What percentage of patients with BCC or SCC develop a second skin cancer within 5 years of the first skin cancer? A. 10% B. 25% C. 50% D. 80% E. 100% 2.After 10 years of immunosuppression, what percentage of transplantation patients develop malignancies? A. 1% B. 10% C. 25% D. 50% E. 80% 3.Which form of skin cancer is associated with the highest risk of simultaneous internal malignancies? A. BCC B. SCC C. Melanoma D. Extramammary Paget's disease E. Bowen's disease 4.Which of the following skin malignancies causes the highest number of deaths each year? A. Squamous cell carcinoma B. Basal cell carcinoma C. Merkel cell carcinoma D. Melanoma
  • 45. Chapter 34: Bone Tumors 1.Factors that limit local recurrence in low-grade extremity soft tissue sarcoma include all of the following except: A.Complete local resection B. Histologically negative margins C. Adjuvant external beam radiation therapy D. Adjuvant brachytherapy E. Primary presentation 2.Risk factors for local recurrence in extremity soft tissue sarcoma include all of the following except: A. Positive microscopic margin B. Fibrosarcoma histopathology C. Deep location D. High histologic grade E. Previous recurrence 3.Risk factors for distant metastasis in extremity soft tissue sarcoma include all of the following except: A. Recurrent presentation B. Size = 10 cm C. Deep location D. Fibrosarcoma histopathology E. High histologic grade 4.True statements about soft tissue sarcomas include the following: A. Approximately 50% occur in the extremities. B. Prior radiation therapy is a causative agent. C. Lymphedema is a predisposing factor. D. Liposarcoma is the most common histopathology E. All of the above 5.Which one of the following is an important factor indicating poor prognosis in extremity soft tissue sarcoma? A. High histologic grade B. Liposarcoma histopathology C. Size < 8 cm D. Distal limb site E. Prior incisional biopsy 6.Biopsy of a primary bone tumor should be done: A. As soon as the lesion is discovered B. After thorough staging studies are completed C. In the operating room D. At the same time that a venous access port is placed
  • 46. 7.Which of the following bone tumors are radiographic diagnoses and do not require biopsy? A. Chondrosarcoma B. Metastasis C. Giant cell tumor D. Osteochondroma 8.Impending fractures should be treated by: A. Radiation therapy B. Chemotherapy C. Internal fixation D. Bisphosphonates 9.Preoperative chemotherapy for osteogenic sarcoma is: A. Needed to perform limb-preserving surgery B. Predictive of disease-free survival C. Useful to tailor postoperative chemotherapy D. Determined by the translocation type causing the tumor 10.Joint replacement to reconstruct defects after a tumor resection is: A. As successful as after removal of an arthritic joint B. Not possible in children younger than 10 years old C. Best for benign tumor cases D. Can be combined with allograft bone transplantation
  • 47. Chapter 35: Head and Neck 1.Which of the following statements regarding aerodigestive tract cancer associated with human papillomavirus (HPV) is false? A. The incidence of HPV-associated aerodigestive tract cancer specific to the tonsil and tongue base subsites is increasing in North America. B. HPV-related aerodigestive tract cancer tends to occur in older patients compared with non—HPV-related aerodigestive tract cancers. C. The incidence of HPV-related aerodigestive tract cancers is increasing at a higher rate in nonsmokers compared with smokers. D. Theincidence of HPV-related aerodigestive tract cancers is increasing at a higher rate in non—alcohol abusers compared with alcohol abusers. 2.A neck dissection that resects nodal levels I through V but preserves the sternocleidomastoid muscle, the spinal accessory nerve, and the internal jugular vein is referred to as a: A. Selective neck dissection B. Modified neck dissection C. Radical neck dissection D. Functional neck dissection 3.To reduce the chance of a vocal fold paralysis postoperatively, the preferred approach to the cervical spine should be: A. Left-sided because it reduces the tension on the recurrent laryngeal nerve during the exposure B. Left-sided because of the incidence of nonrecurrent recurrent laryngeal nerve on the right side C. Right-sided because it reduces the tension on the recurrent laryngeal nerve during the exposure D. Right-sided because of the incidence of nonrecurrent recurrent laryngeal nerve on the left side 4.Why should percutaneous tracheotomy performed in the ICU be avoided in patients with prolonged transoral intubation secondary to ventilator dependence? A. Percutaneous tracheotomy has a higher incidence of postdecannulation stenosis in patients undergoing tracheotomy owing to failure to wean from mechanical ventilation. B. Percutaneous tracheotomy has a higher incidence of complications from accidental decannulation because an inferiorly based trachea-to-skin flap specific to this procedure is not created. C. Percutaneous tracheotomy has a higher incidence of "false lumen" creation than traditional open tracheotomy performed in the operating room. D. Percutaneous tracheotomy does not have a higher incidence of intraoperative or postoperative complications and is as safe to perform in ventilator-dependent patients as traditional open tracheotomy
  • 48. 5.An adult patient presents with an asymptomatic, solitary 4-cm neck mass that has been present for 1 month. Physical examination and history are otherwise unremarkable. The initial step in the workup for this mass would be: A. CT scan with intravenous contrast agent B. Open incisional biopsy C. Fine-needle aspiration D. Open excisional biopsy with conversion to neck dissection depending on intraoperative frozen section diagnosis 6.A patient presents with a glottic squamous cell carcinoma involving both anterior true vocal cords but not either of the arytenoid cartilages, and is not a candidate for endoscopic surgical treatment because of the inability to expose the larynx for laser excision. A viable conservation surgical therapy would be: A. Supraglottic laryngectomy B. Supracricoid laryngectomy with cricohyoidoepiglottopexy C. Vertical partial laryngectomy D. Total laryngectomy with tracheoesophageal puncture 7. A 30-year-old woman presents with a slowly growing, asymptomatic, 3-cm parotid mass. Office fine- needle aspiration is inconclusive. Assuming that the intraoperative frozen section shows this mass to be a benign salivary neoplasm, the most appropriate surgical plan would be to perform: A. Incisional biopsy without the need for further surgery B. Total parotidectomy with facial nerve dissection and preservation C. Excisional biopsy without the need for further surgery D. Superficial parotidectomy with facial nerve dissection and preservation 8.A patient has a recurrent laryngeal nerve resection as part of removal of an aggressive thyroid malignancy. Postoperatively, her voice is breathy and weak and is insufficient for performing at her profession. The goal of subsequent voice restoration surgery is to: A. Create volitional abduction and adduction of the affected vocal cord to preserve both voice and airway patency B. Create volitional abduction and adduction of the affected vocal cord to preserve both voice and airway protection from aspiration during swallowing C. Medialize the affected vocal cord to create static contact with the opposite, mobile cord D. Place a Silastic implant that is capable of directly contacting the opposite, mobile cord
  • 49. 9.An adult patient presents with a right-sided, 3-cm asymptomatic neck mass deep to the sternocleidomastoid muscle at the level of the hyoid bone that has been growing over the past 3 months. The most likely etiology in this patient is: A. Branchial cleft cyst B. Malignant lymphadenopathy C. Lipoma D. Carotid body tumor 10.The level of the neck nodes bounded by the laryngeal strap muscles anteriorly, the posterior border of the sternocleidomastoid muscle posteriorly, a horizontal plane at the level of the cricoid cartilage inferiorly, and the level of the hyoid bone superiorly is referred to as level: A. Ia B. II C. III D. IV
  • 50. Chapter 37: Breast Reconstruction 1.A 43-year-old woman is scheduled for reconstruction of the right breast with a latissimus dorsi flap. She is at greatest risk for which of the following complications? A. Chronic chest wall pain B. Dorsal wound dehiscence C. Partial flap necrosis D. Seroma E. Stiffness in the ipsilateral shoulder 2.A 58-year-old nulligravid woman who is scheduled to undergo bilateral prophylactic mastectomy comes to the office for consultation regarding immediate breast reconstruction. She works full time as a fitness instructor. The patient currently wears a size 34B brassiere and wants her bra size to be increased to a C cup, but she wants to make sure that scarring is minimized. Her height is 5 feet, 5 inches and weight is 120 lb. Physical examination of the abdomen shows a paucity of extra tissue. Which of the following is the most appropriate breast reconstruction procedure for this patient? A. Bilateral autogenous reconstruction B. Bilateral tissue expansion followed by implantation of prostheses C. Delayed breast reconstruction after pathology is confirmed D. Single-stage reconstruction with prostheses E. TRAM flap followed by implantation of prostheses 3.A 55-year old woman undergoes a modified radical mastectomy with immediate first-stage reconstruction of the right breast with a tissue expander. Before beginning the second stage, to exchange the tissue expander with a permanent prosthesis, pathology results from analysis of tissue from the right breast indicate metastatic carcinoma of four axillary lymph nodes. Radiation therapy is recommended. Which of the following interventions will result in the best long-term appearance of the reconstructed breast? A. Complete the tissue expansion before radiation and exchange the tissue expander with a prosthesis after radiation. B. Deflate the tissue expander before radiation; reinflate the tissue expander and exchange with a prosthesis after radiation. C. Remove the tissue expander and reconstruct the breast with a TRAM flap before radiation. D. Remove the tissue expander before radiation; after radiation, reinsert and expand a tissue expander and then exchange with a prosthesis. E. Remove the tissue expander before radiation and reconstruct the breast with a TRAM flap after radiation 4.The skin-sparing mastectomy involves the preservation of the: A. Areola B. Nipple C. Inframammary crease D. Skin E. Skin and areola
  • 51. 5.A 45-year-old woman with T3N0 invasive ductal carcinoma in the inferior pole of the left breast is scheduled to undergo segmental mastectomy and subsequent radiation therapy. She currently wears a size 36DDD bra and is willing to accept any cup size from C to DDD. Which of the following interventions will yield the best cosmetic result in this patient? A. Bilateral reduction mammaplasty B. Implantation of a prosthesis in the left breast and mastopexy of the right breast C. Latissimus dorsi musculocutaneous flap reconstruction of the left breast and mastopexy of the right breast D. TRAM reconstruction of the defect E. No reconstruction 6.A 40-year-old woman is scheduled to undergo reconstruction of the right breast via a free TRAM flap. She has smoked two packs of cigarettes daily for the past 8 years. This patient's smoking history increases her risk of which of the following postoperative complications? A. Hematoma B. Mastectomy flap necrosis C. Seroma D. TRAM flap loss E. Vessel thrombosis 7.A 48-year-old woman comes to the office for consultation regarding reconstruction of her right breast after mastectomy because of cancer. The patient is concerned about maximizing the aesthetic result and minimizing any donor site deformity. Physical examination shows a well-healed chest wall and a B cup left breast with grade 3 ptosis. Soft tissue reconstruction with an SGAP free flap is planned. Which of the following is a disadvantage of this procedure? A. Difficulty molding the gluteal fat B. Gait dysfunction C. Inability to hide the donor scar completely D. Inability to provide a sensate flap E. Lack of abundant soft tissue 8.A healthy 27-year-old woman comes to the office for follow-up examination 3 months after undergoing bilateral prophylactic mastectomy and reconstruction with saline prostheses. She is concerned about the incision on her left breast because it is slightly swollen and warm to the touch. Physical examination shows mild erythema of the left breast and normal healing of the right breast. Which of the following is the most appropriate initial management? A. Observation B. Oral antibiotic therapy C. Open lavage of the implant pocket D. Removal of the prosthesis E. Exchange of the prosthesis
  • 52. 9.A 36-year-old woman comes to the office for consultation regarding breast reconstruction 1 year after undergoing a right modified radical mastectomy. The procedure was followed by 6 weeks of radiation therapy. She has no history of other surgical procedures or serious medical illnesses. She is 5 feet. 4 inches in height and weighs 135 lb. She wears a size 32B brassiere. Which of the following is the least appropriate breast reconstruction procedure for this patient? A. Extended latissimus dorsi flap B. Latissimus dorsi flap with saline-filled prosthesis C. SGAP flap D. TRAM flap E. Two-stage reconstruction with a tissue expander and saline-filled prosthesis 10.Secondary breast reconstruction procedures such as nipple reconstruction and areolar tattooing are considered: A. At time of mastectomy and immediate breast reconstruction B. Prior to the patient receiving chemotherapy C. Prior to the patient receiving radiotherapy D. After chemotherapy and radiation therapy E. Prior to flap or expander placement
  • 53. Chapter 38: Thyroid 1.The recurrent laryngeal nerve provides motor innervation to all muscles of the larynx. A. The recurrent laryngeal nerve provides motor innervation to all muscles of the larynx. B. The thyroid forms as two lateral projections from the developing alimentary tract that fuse in the midline during development. C. Unilateral vocal cord paralysis may have minimal clinical symptoms. D. All cells in the adult thyroid are of endodermal origin. 2.You evaluate a 35-year-old woman with a palpable nodule in the right lateral neck. No other lateral neck masses are appreciable on examination. Fine-needle aspiration biopsy of this mass reveals thyroid cells. Which of the following statements is true? A. In the absence of a palpable thyroid nodule, this likely represents lateral aberrant thyroid, which is a normal embryologic variant. B. When found in the setting of the most common pathologic type of thyroid cancer, this finding does not significantly alter the expected survival for this patient. C. Given the pathologic finding, the appropriate surgical approach is total thyroidectomy and selective excision of all lymph nodes that appear suspicious on operative evaluation. D. In the absence of palpable adenopathy, it would be unlikely to find pathologic evidence of metastasis to regional lymph nodes from the most common type of thyroid cancer. 3.You evaluate a 27-year-old woman with a thyroid nodule. Fine-needle aspiration biopsy is consistent with medullary thyroid cancer. Which of the following statements is true? A. Most cases of medullary thyroid cancer are sporadic and not associated with a familial syndrome. B. The next appropriate step in management is to proceed directly to total thyroidectomy with bilateral central compartment lymph node dissection. C. In a case of the finding of RET proto-oncogene mutation in a patient with no palpable or ultrasound- detectable thyroid nodule, annual observation is indicated, and thyroidectomy should be performed when a nodule is first appreciated on examination or imaging. D. After resection for medullary thyroid cancer and recovery from surgery, radioiodine ablation with either thyroid hormone withdrawal or thyroid-stimulating hormone (TSH) administration is the next therapeutic intervention. 4.All of the following statements regarding the treatment of papillary thyroid cancer are true except which one (which is false)? A. Total thyroidectomy is indicated for cancers greater than 1 cm in greatest dimension and for smaller cancers with adverse features such as a history of radiation exposure. B. Reoperation is indicated for completion thyroidectomy for patients who have a papillary thyroid cancer greater than 1 cm discovered on permanent section pathology after thyroid lobectomy. C. Prophylactic lateral compartment neck dissection (ipsilateral modified radical neck dissection) is indicated in most cases. D. After resection, exogenous thyroid hormone should be titrated to achieve a subnormal thyroid-stimulating hormone (TSH).
  • 54. 5.All of the following statements are characteristic of hormones produced by the thyroid gland except which one (which is false)? A. Iodine is essential for the production of thyroid hormones. B. The enzyme thyroid peroxidase is an integral part of thyroid hormone production. C. Thyroglobulin (Tg) is the storage form of thyroid hormone. D. Thyroid hormone production and release are predominantly regulated by thyroid-stimulating hormone (TSH) from the pituitary gland. E. Most hormone released by the thyroid is triiodothyronine (T3). 6.Which of the following statements is true? A. Thyroglobulin (Tg) has predictive value for the recurrence of well-differentiated thyroid cancer. B. After surgery, calcitonin is replaced by a once-daily oral regimen to maintain calcium homeostasis. C. Thyroid-stimulating hormone (TSH) levels should be maintained in the high to normal range after surgery for well-differentiated thyroid cancer. D. Corticosteroids stimulate thyroid hormone release and enhance peripheral conversion of thyroxine (T4) to triiodothyronine (T3). E. All statements are true. 7.Complete surgical resection (thyroidectomy) is first-line therapy for which one of the following? A. Riedel thyroiditis (struma) B. Toxic nodular goiter with marked compressive symptoms C. Acute suppurative thyroiditis D. Uncomplicated Graves disease (diffuse toxic goiter) E. Hashimoto thyroiditis 8.You are preparing a patient for total thyroidectomy for treatment of Graves disease (diffuse toxic goiter). To attempt to avoid complications from severe thyrotoxicosis, including life-threatening thyroid storm, you could employ any of the following therapies except which one (which is not appropriate)? A. Beta blockade with an agent such as propranolol B. Large doses of thionamides such as propylthiouracil (PTU) and methimazole C. Large doses of iodine after a thionamide D. Rapid fluid replacement along with corticosteroids E. Large doses of amiodarone 9.Which of the following is associated with an aberrant takeoff of the right subclavian artery? A. Nonrecurrent left laryngeal nerve B. Absence of the right thyroid lobe C. Absence of the thyroid isthmus D. Nonrecurrent right laryngeal nerve E. Nonrecurrent right external branch of the superior laryngeal nerve
  • 55. 10.A palpable lymph node is found along the posterior-lateral border of the anterior belly of the digastric muscle. This node lies in which anatomic lymph node basin? A. IA B. IB C. IIA D. IIB E. VI
  • 56. Chapter 39: The Parathyroid Glands 1.Embryologic aspects of the superior parathyroid gland include which of the following? A. Origin from branchial pouch III B. Origin from branchial pouch V C. Typically associated with the thyroidal tubercle of Zuckerkandl D. Frequent ectopic location within thyroid parenchyma 2.Normally embedded in fat and located within a 2-cm circumscribed area that is cranial to the intersection of the inferior thyroid artery and recurrent laryngeal nerve is a description that applies to the: A. Superior parathyroid gland B. Inferior parathyroid gland C. Pyramidal lobe of thyroid gland D. Thymus 3.Which of the following is true regarding human parathyroid hormone? A. Intact parathyroid hormone is 92 amino acids long. B. Parathyroid hormone is principally metabolized in the kidney. C. Biologically active parathyroid hormone fragments include the C-terminal sequence of the intact molecule. D. Parathyroid hormone secretion is closely linked with the intracellular concentration of ionized calcium. 4.An adult with a serum calcium concentration of 10.6 mg/dL (normal, 8.6 to 10.2 mg/dL), serum intact PTH level of 90 pg/mL (normal, 10 to 65 pg/mL), and a 24-hour urinary calcium value of 25 mg/day has: A. Secondary hyperparathyroidism B. Tertiary hyperparathyroidism C. A defect in the calcium-sensing receptor gene D. A syndrome successfully treated with surgery 5.Parathyroid carcinoma: A. Is most common at the extremes of age B. Is often associated with mild hypercalcemia C. Is optimally treated with en bloc resection of the ipsilateral thyroid lobe at the initial operation D. Is always easy to recognize at the time of operation 6.Patients with asymptomatic primary hyperparathyroidism should undergo parathyroidectomy when: A. The serum calcium concentration is 1 mg/dL above the upper limit of normal. B. They are unwilling to undergo medical surveillance. C. They are younger than 50 years. D. The T-score is less than –2.5 on bone mineral density measurement. E. All of the above
  • 57. 7.Ectopic locations for superior parathyroid adenomas include: A. In the tracheoesophageal groove near the esophagus B. In a retroesophageal plane in the upper posterior mediastinum C. Undescended near the submandibular gland D. All of the above 8.Which of the following is correct about intraoperative parathyroid hormone monitoring? A. Quick intraoperative chemiluminescent assays measure only the N-terminal fragment of PTH. B. The most commonly used criterion for cure is a 25% decrease from baseline at 10 minutes postexcision. C. Monitoring is practical because intact PTH has a very short half-life. D. The value is never affected by aggressive dissection of the adenoma. 9.Common reasons for persistent hyperparathyroidism after initial parathyroidectomy include: A. Failure to find the causative adenoma at the initial operation B. Failure to detect or recognize a second adenoma (double adenoma) C. Failure to recognize and aggressively treat multigland hyperplasia D. All of the above 10.Parathyroidectomy for secondary hyperparathyroidism is indicated when: A. Patients have refractory bone or joint pain and muscular weakness. B. Calciphylaxis occurs. C. Tertiary hyperparathyroidism is evident. D. All of the above
  • 58. Chapter 40: Endocrine Pancreas 1. The endocrine tumor of the pancreas with the lowest rate of malignancy is: A. Glucagonoma B. Somatostatinoma C. Gastrinoma D. Insulinoma 2. Which of the following actions is not performed by insulin? A. Decreasing blood sugar level B. Decreasing protein synthesis C. Decreasing glycogenolysis D. Decreasing lipolysis E. Increasing glucose transport 3. The best means to localize a gastrinoma preoperatively is: A. Enhanced MRI B. Selective portal venous sampling C. Calcium angiography D. Somatostatin receptor scintigraphy E. Endoscopic ultrasound 4. In patients with severe diarrhea, continuous nasogastric aspiration may provide a diagnostic clue for: A. Sprue B. Zollinger-Ellison syndrome C. Cholera D. Verner-Morrison (VIPoma) syndrome E. Glucagonoma 5. Which of the following endocrine tumors of the pancreas has the best prognosis for cure? A. Insulinoma B. Somatostatinoma C. VIPoma D. Glucagonoma E. Gastrinoma
  • 59. 6. The most effective intraoperative technique for localizing a pancreatic gastrinoma involves palpation plus: A. Intraoperative endoscopy with transillumination B. Selective venous sampling C. Intra-arterial injection of vital blue dye D. Intraoperative ultrasonography E. Caudal pancreatectomy 7. MEN1 syndrome is usually associated with tumors in all but which of the following glands? A. Thyroid B. Adrenal C. Pituitary D. Pancreas E. Parathyroid 8. The most accurate method for localizing an insulinoma before operation is: A.MRI B. CT C. Somatostatin receptor scintigraphy D. Selective angiography E. Intra-arterial calcium stimulation 9. Clinical clues to Zollinger-Ellison syndrome include all the following except: A. Hypercalcemia B. Gastroesophageal reflux disease (GERD) C. A positive corticotropin test D. Diarrhea E. High output of gastric acid 10. The most common site for gastrinomas is the: A. Head of pancreas in gastrinoma triangle B. Prepyloric area C. Body and tail of pancreas D. Duodenum E. Gastroduodenal ligament
  • 60. 11. The salient clinical characteristic of the glucagonoma syndrome is: A. Brittle hyperglycemia B. High serum calcium C. Necrolytic migrating erythema D. Secretory diarrhea E. Massive fatty infiltration of the liver
  • 61. Chapter 43: Esophagus 1. Which of the following statements about esophageal anatomy is correct? A. The esophagus has a poor blood supply, which is segmental in distribution and accounts for the high incidence of anastomotic leakage. B. The esophageal serosa consists of a thin layer of fibroareolar tissue. C. The esophagus has two distinct muscle layers—an outer, longitudinal and an inner, circular layer—that are striated in the upper third and smooth in the distal two thirds. D. Injury to the recurrent laryngeal nerve results in vocal cord dysfunction, but does not affect swallowing. E. The lymphatic drainage of the esophagus is sparse, localized primarily to adjacent paraesophageal lymph nodes. 2. Which of the following statements about achalasia is correct? A. In most cases in North America, the cause is a parasitic infestation by Trypanosoma cruzi. B. Chest pain and reflux are the usual symptoms. C. Distal third esophageal adenocarcinomas may occur in 30% of patients within 5 years of diagnosis. D. Manometry shows failure of LES relaxation on swallowing and absent or weak simultaneous contractions in the esophageal body after swallowing. E. Endoscopic botulinum toxin injection of the LES, pneumatic dilation, and esophagomyotomy can provide highly effective curative therapy for achalasia. 3. Which of the following statements regarding the pathology of esophageal carcinoma is correct? A. Worldwide, adenocarcinoma is the most common esophageal malignancy. B. Squamous cell carcinoma is most common in the distal esophagus, whereas adenocarcinoma predominates in the middle third. C. Patients with Barrett's metaplasia are more likely than the general population to develop adenocarcinoma. D. Metastases from esophageal carcinoma are characteristically limited to regional mediastinal l lymph nodes adjacent to the tumor. E. Achalasia, radiation esophagitis, caustic esophageal stricture, and Plummer-Vinson syndrome are premalignant esophageal lesions that predispose to the development of adenocarcinoma.
  • 62. 4. Which of the following statements about the surgical treatment of esophageal carcinoma is correct? A. The finding of severe dysplasia in association with Barrett's mucosa is an indication for an antireflux operation to prevent subsequent development of carcinoma. B. Long-term survival at all stages is improved by radical en bloc resection of the esophagus with its contained tumor, adjacent mediastinal tissues, and regional lymph nodes. C. The morbidity and mortality rates for cervical esophagogastric anastomotic leak are lower than the rates associated with intrathoracic esophagogastric anastomotic leak. D. The leading complications of transthoracic esophagectomy and intrathoracic esophagogastric anastomosis are bleeding and wound infection. E. Transhiatal esophagectomy without thoracotomy achieves better long-term survival than transthoracic esophagectomy. 5. The best management for a 48-hour-old distal esophageal perforation is: A. Antibiotics and drainage B. Division of the esophagus and exclusion of the perforation C. Primary repair with buttressing D. Resection with cervical esophagostomy, gastrostomy, and jejunostomy E. T-tube fistula and drainage 6. A 42-year-old man with a history of alcoholism and tobacco use is diagnosed by barium swallow with an esophageal stricture 10 cm from the gastroesophageal junction. Esophagoscopy confirms a moderate stricture, and biopsy reveals fibrosis. What is the next step in treatment? A. Diet alteration and acid suppression with H2 inhibitors B. Endoscopic stricture dilation and acid supression with high-dose proton pump inhibitor (PPI) therapy C. Fluoroscopy-guided stent placement D. Esophagectomy with gastric pull-through E. Laparoscopic fundoplication 7. The most common cause of morbidity after esophagectomy is: A. Postoperative hemorrhage B. Pneumonia C. Anastomotic leak D. Recurrent laryngeal nerve injury E. Chylothorax
  • 63. 8. A 54-year-old woman has severe chest pain intermittently after meals. An extensive cardiac workup has been negative. Esophageal manometry before and after swallowing a 5-mL water bolus shows lower esophageal sphincter resting pressure of 30 mm Hg, length of 3 cm, and residual pressure of 5 mm Hg. Peristaltic contractions have an amplitude of 204 mm Hg 5 cm above the lower esophageal sphincter and a duration of 7 seconds. These measurements are most consistent with: A. Nutcracker esophagus B. Achalasia C. Diffuse esophageal spasm D. Nonspecific esophageal dysmotility E. Hypertensive lower esophageal sphincter 9. The most crucial step in the management of a patient with Zenker's diverticulum is to: A. Identify the underlying motility disorder preoperatively B. Perform a complete cricopharyngotomy at operation C. Resect the diverticulum at operation D. Drain the surgical site postoperatively E. Perform an antireflux procedure at the time of surgery 10.Leiomyomas of the esophagus: A. Require esophagectomy for definitive treatment because of submucosal spread B. Appear as a distinct rough-edged mass on barium swallow C. Account for <25% of benign esophageal tumors D. Do not require biopsy for preoperative diagnosis E. Metastasize primarily to supraclavicular nodes
  • 64. Chapter 44: Hiatal Hernia and Gastroesophageal Reflux Disease 1. Components of the lower esophageal sphincter include all of the following except: A. Sling fibers of the cardia B. Striated external sphincter muscle fibers C. Diaphragm D. Intrinsic muscle of the distal esophagus E. Transmitted pressure of the abdominal cavity 2. A partial fundoplication (<360 degrees) should always be performed in patients with ineffective esophageal motility (IEM). A. True B. False 3. After antireflux surgery: A. Short-term bloating occurs in more than 60% of patients B. Long-term dysphagia is present in 50% of patients C. Dilation should be performed for any dysphagia in the first 2 months D. Long-term bloating may be the direct result of vagal nerve injury 4. Which of the following statements concerning extraesophageal manifestations of GERD is true? A. In some patients, GERD is manifested only with extraesophageal symptoms (i.e., these patients have no heartburn). B. Diagnosing the association of GERD with extraesophageal symptoms is always possible if all diagnostic tools are used. C. Extraesophageal symptoms respond to medical and surgical antireflux therapy to the same degree as heartburn. D. In patients with GERD, extraesophageal symptoms are caused by vagal-mediated pathways. 5. Paraesophageal hernias: A. Have a high likelihood of acute strangulation, and all should be repaired when identified even if asymptomatic B. Are rarely associated with chronic gastrointestinal blood loss C. Should be repaired only in patients with symptoms D. Are best diagnosed by endoscopy
  • 65. Chapter 45: Abdominal Wall, Umbilicus, Peritoneum, Mesenteries, Omentum, and Retroperitoneum 1. Which of the following statements is true regarding the anatomy of the abdominal wall? A. The inguinal or Poupart's ligament is the lower free edge of the aponeurosis of the transversus abdominis muscle. B. The rectus sheath is composed of aponeurotic fibers from the internal and external oblique and transversus abdominis muscles. C. The rectus sheath completely envelops the anterior and posterior surfaces of the rectus a abdominis muscle from its origin in the costal cartilages to its insertion on the pubis. D. The lymphatic vessels of the abdominal wall drain almost exclusively into the superficial and deep inguinal lymph node chains. E. The abdominal wall is innervated almost exclusively from branches of the first through fourth lumbar spinal nerves. 2. By definition, a hernia results from a defect in which of the following structures? A. Transversalis fascia B. Peritoneum C. Internal oblique fascia D. Internal and external oblique fascia E. Rectus sheath 3. Which of the following are not contained within the preperitoneal space? A. The medial umbilical ligaments, which are the remnants of the obliterated umbilical arteries B. The median umbilical ligament, which is the remnant of the obliterated allantoic stalk (or urachus) C. The inferior epigastric arteries and veins D. The superior epigastric arteries and veins E. The falciform ligament containing the round ligament of the liver, which is the remnant of the obliterated umbilical vein 4. All the following are abnormalities related to persistence of the omphalomesenteric duct except: A. Meckel's diverticulum B. Omphalocele C. Enterocutaneous fistula D. Umbilical sinus E. Intestinal obstruction from volvulus
  • 66. 5. All the following factors have been related to the development of rectus sheath hematomas except: A. Pregnancy B. Strenuous muscular exertion C. Anticoagulation D. Benign and malignant tumors of the abdominal wall E. Trauma 6. All the following statements are true regarding the natural history of desmoid tumors of the abdominal wall except: A. Patients with familial adenomatous polyposis syndrome have a much higher incidence of desmoid tumors than does the general population. B. Tumors manifest as painless enlarging masses, often with a relatively rapid rate of growth. C. Most patients have systemic metastases to the liver and lung at the time of presentation. D. The development of abdominal wall desmoid tumors is often temporally related to a recent childbirth or abdominal operation or trauma. 7. The clinical features of an abdominal wall mass that should suggest a malignant cause include all the following except: A. The occurrence of severe unrelenting pain in an abdominal wall mass B. Nonreducible masses arising from below the superficial fascia C. Size > 5 cm D. A recent increase in size E. Fixation to the abdominal wall or underlying organs 8. The peritoneum and peritoneal cavity respond to infection by all the following except: A. The rapid clearance of particulate matter, including bacteria, via the normal circulation of peritoneal fluid toward and ultimately into the diaphragmatic lymphatic channels and thoracic duct. B. The generation of potent anti-inflammatory mediators by peritoneal macrophages, which prevent the movement of leukocytes into the peritoneal cavity from the surrounding microvasculature C. The release of histamine and other vasoactive products by degranulating peritoneal mast cells, causing the extravasation of protein-rich fluid into the peritoneal space D. The opsonization of bacteria by proteins within the fluid generated by the inflamed peritoneal Membrane E. The sequestration of bacteria within fibrin matrices, promoting abscess formation and limiting the generalized spread of infection throughout the peritoneal cavity
  • 67. 9. Which of the following are important causes of ascites? A. Cirrhosis B. Peritoneal carcinomatosis C. Traumatic injuries to the retroperitoneal lymphatic channels D. Nephrotic syndrome E. All of the above 10. All the following are valuable diagnostic measures in the evaluation of cirrhotics with new-onset ascites except: A. Physical examination showing a full bulging abdomen, with dullness of the flanks on percussion B. Paracentesis with measurement of ascitic fluid cell count and differential C. Paracentesis with measurement of ascitic fluid albumin concentration D. Determination of the serum-albumin to ascites-albumin gradient (SAAG) E. Paracentesis with Gram staining of the ascitic fluid 11. All the following statements are true regarding spontaneous bacterial peritonitis (SBP) except: A. SBP occurs almost exclusively in patients with high protein concentrations in the ascitic fluid (i.e., SAAG < 1.1 g/dL). B. The most common pathogens in adults are Escherichia coli and Klebsiella pneumoniae, whereas in children streptococcal species and Staphylococcus aureus play a greater role. C. The diagnosis of SBP is made initially by showing >250 neutrophils/mm3 of ascitic fluid in the setting of abdominal pain, fever, or peripheral leukocytosis or some combination of these. D. Gram staining of ascitic fluid usually does not show bacterascites. 12. All the following statements are true regarding intra-abdominal hernias except: A. Mesocolic (or paraduodenal) hernias result from abnormalities of embryonic midgut rotation. B. The superior mesenteric artery and vein course along the medial border of the neck of right and left mesocolic hernias. C. Patients with intra-abdominal hernias present most commonly with signs and symptoms of intestinal obstruction. D. Internal hernias may result from the creation of mesenteric windows at the time of bowel resections. E. All of the above are true.
  • 68. 13. All the following are advantages to a retroperitoneal operative approach compared with a transabdominal approach to retroperitoneal organs except: A. Less postoperative ileus with a more rapid resumption of a normal diet B. Less operative time and blood loss C. No intra-abdominal adhesions, reducing the likelihood of subsequent small bowel obstructions D. Less intraoperative evaporative fluid losses with less dramatic intravascular fluid shifts E. Fewer respiratory complications, such as atelectasis or pneumonia
  • 69. Chapter 46: Hernias 1. The best outcome for recurrent inguinal hernia repair is achieved by: A. Selecting the same approach used for the initial repair B. Experienced hernia surgeons at hernia centers C. Cooper ligament repair D. Using prosthetic mesh E. Bassini repair 2. Which of the following is not a common cause of inguinal hernia recurrence? A. Excessive suture line tension B. Failure to recognize an indirect hernia C. Increased intra-abdominal pressure D. Failure to a use tension-free or mesh repair E. Poor collagen formation 3. Failure to recognize which of the following during repair of a sliding inguinal hernia is associated with increased complications? A. The colon usually constitutes part of the hernia sac. B. Sliding hernias are usually direct hernias. C. A separate incision is necessary for reduction. D. Tension-free repair should be avoided. E. High ligation of the sac is usually necessary. 4. Femoral hernias are: A. The most common type of hernia in women B. Usually bilateral C. Strangulated in 15% to 20% of cases D. More common on the left side E. Associated with an inguinal hernia in >50% of patients 5. Nerve injury during laparoscopic hernia repair is best avoided by not placing tacks: A. Medial to the pubic tubercle B. Superior to the lateral iliopubic tract C. Inferior to the lateral iliopubic tract D. In the Cooper ligament E. In the peritoneum